MRCS preparation emrcs questions upperlimb

206
A 43 year old man is stabbed outside a nightclub. He suffers a transection of his median nerve just as it leaves the brachial plexus. Which of the following features is least likely to ensue? A. Ulnar deviation of the wrist B. Complete loss of wrist flexion C. Loss of pronation D. Loss of flexion at the thumb joint E. Inability to oppose the thumb Next question Loss of the median nerve will result in loss of function of the flexor muscles. However, flexor carpi ulnaris will still function and produce ulnar deviation and some residual wrist flexion. High median nerve lesions result in complete loss of flexion at the thumb joint. Median nerve The median nerve is formed by the union of a lateral and medial root respectively from the lateral (C5,6,7) and medial (C8 and T1) cords of the brachial plexus; the medial root passes anterior to the third part of the axillary artery. The nerve descends lateral to the brachial artery, crosses to its medial side (usually passing anterior to the artery). It passes deep to the bicipital aponeurosis and the median cubital vein at the elbow. It passes between the two heads of the pronator teres muscle, and runs on the deep surface of flexor digitorum superficialis (within its fascial sheath). Near the wrist it becomes superficial between the tendons of flexor digitorum superficialis and flexor carpi radialis, deep to palmaris longus tendon. It passes deep to the flexor retinaculum to enter the palm, but lies anterior to the long flexor tendons within the carpal tunnel. Branches Region Branch Upper arm No branches, although the nerve commonly communicates with the musculocutaneous nerve Forearm Pronator teres Flexor carpi radialis Palmaris longus Flexor digitorum superficialis Flexor pollicis longus Flexor digitorum profundus (only the radial half) Distal forearm Palmar cutaneous branch Hand (Motor) Motor supply (LOAF) Lateral 2 lumbricals Opponens pollicis Abductor pollicis brevis Flexor pollicis brevis Hand (Sensory) Over thumb and lateral 2 ½ fingers On the palmar aspect this projects proximally, on the dorsal aspect only the distal regions are innervated with the radial nerve providing the more proximal cutaneous innervation.

Transcript of MRCS preparation emrcs questions upperlimb

Page 1: MRCS preparation emrcs questions upperlimb

A 43 year old man is stabbed outside a nightclub. He suffers a transection of his median nerve just as it leaves the brachial plexus.

Which of the following features is least likely to ensue?

A. Ulnar deviation of the wrist

B. Complete loss of wrist flexion

C. Loss of pronation

D. Loss of flexion at the thumb joint

E. Inability to oppose the thumb

Next question

Loss of the median nerve will result in loss of function of the flexor muscles. However, flexor carpi ulnaris will still function and produce

ulnar deviation and some residual wrist flexion. High median nerve lesions result in complete loss of flexion at the thumb joint.

Median nerve

The median nerve is formed by the union of a lateral and medial root respectively from the lateral (C5,6,7) and medial (C8 and T1)

cords of the brachial plexus; the medial root passes anterior to the third part of the axillary artery. The nerve descends lateral to the

brachial artery, crosses to its medial side (usually passing anterior to the artery). It passes deep to the bicipital aponeurosis and the

median cubital vein at the elbow.

It passes between the two heads of the pronator teres muscle, and runs on the deep surface of flexor digitorum superficialis (within its

fascial sheath).

Near the wrist it becomes superficial between the tendons of flexor digitorum superficialis and flexor carpi radialis, deep to palmaris

longus tendon. It passes deep to the flexor retinaculum to enter the palm, but lies anterior to the long flexor tendons within the carpal

tunnel.

Branches

Region Branch

Upper arm No branches, although the nerve commonly communicates with the musculocutaneous nerve

Forearm Pronator teres

Flexor carpi radialis

Palmaris longus

Flexor digitorum superficialis

Flexor pollicis longus

Flexor digitorum profundus (only the radial half)

Distal

forearm

Palmar cutaneous branch

Hand (Motor) Motor supply (LOAF)

Lateral 2 lumbricals

Opponens pollicis

Abductor pollicis brevis

Flexor pollicis brevis

Hand

(Sensory) Over thumb and lateral 2 ½ fingers

On the palmar aspect this projects proximally, on the dorsal aspect only the distal regions are innervated with the radial nerve

providing the more proximal cutaneous innervation.

Page 2: MRCS preparation emrcs questions upperlimb

Patterns of damage

Damage at wrist

e.g. carpal tunnel syndrome

paralysis and wasting of thenar eminence muscles and opponens pollicis (ape hand deformity)

sensory loss to palmar aspect of lateral (radial) 2 ½ fingers

Damage at elbow, as above plus:

unable to pronate forearm

weak wrist flexion

ulnar deviation of wrist

Anterior interosseous nerve (branch of median nerve)

leaves just below the elbow

results in loss of pronation of forearm and weakness of long flexors of thumb and index finger

Topography of the median nerve

Page 3: MRCS preparation emrcs questions upperlimb

A 24 year old man falls and sustains a fracture through his scaphoid bone. From which of the following areas does the scaphoid

derive the majority of its blood supply?

A. From its proximal medial border

B. From its proximal lateral border

C. From its proximal posterior surface

D. From the proximal end

E. From the distal end

Next question

Theme from April 2012 Exam

The blood supply to the scaphoid enters from a small non articular surface near its distal end. Transverse fractures through the

scaphoid therefore carry a risk of non union.

Scaphoid bone

The scaphoid has a concave articular surface for the head of the capitate and at the edge of this is a crescentic surface for the

corresponding area on the lunate.

Proximally, it has a wide convex articular surface with the radius. It has a distally sited tubercle that can be palpated. The remaining

articular surface is to the lateral side of the tubercle. It faces laterally and is associated with the trapezium and trapezoid bones.

The narrow strip between the radial and trapezial surfaces and the tubercle gives rise to the radial collateral carpal ligament. The

tubercle receives part of the flexor retinaculum. This area is the only part of the scaphoid that is available for the entry of blood

vessels. It is commonly fractured and avascular necrosis may result.

A 56 year old man requires long term parenteral nutrition

and the decision is made to insert a PICC line for long

term venous access. This is inserted into the basilic vein

at the region of the elbow. As the catheter is advanced,

into which venous structure is the tip of the catheter most

likely to pass from the basilic vein?

A. Subclavian vein

B. Axillary vein

C. Posterior circumflex humeral vein

Page 4: MRCS preparation emrcs questions upperlimb

D. Cephalic vein

E. Superior vena cava

Next question

The basilic vein drains into the axillary vein and although

PICC lines may end up in a variety of fascinating locations

the axillary vein is usually the commonest site following

from the basilic. The posterior circumflex humeral vein is

encountered prior to the axillary vein. However, a PICC

line is unlikely to enter this structure because of its angle

of entry into the basilic vein.

Basilic vein

The basilic and cephalic veins both provide the main

pathways of venous drainage for the arm and hand. It is

continuous with the palmar venous arch distally and the

axillary vein proximally.

Path

Originates on the medial side of the dorsal venous

network of the hand, and passes up the forearm and

arm.

Most of its course is superficial.

Near the region anterior to the cubital fossa the vein

joins the cephalic vein.

Page 5: MRCS preparation emrcs questions upperlimb

Midway up the humerus the basilic vein passes deep

under the muscles.

At the lower border of the teres major muscle, the

anterior and posterior circumflex humeral veins feed

into it.

Joins the brachial veins to form the axillary vein.

Theme: Nerve injury

A. Ulnar nerve

B. Musculocutaneous nerve

C. Radial nerve

D. Median nerve

E. Axillary nerve

F. Intercostobrachial nerve

What is the most likely nerve injury for the scenario given?

Each option may be used once, more than once or not at

all.

16. A 23 year old man is involved in a fight outside a

nightclub and sustains a laceration to his right arm. On

examination he has lost extension of the fingers in his

right hand.

Radial nerve

The radial nerve supplies the extensor muscle group.

Page 6: MRCS preparation emrcs questions upperlimb

17. A 40 year old lady trips and falls through a glass door

and sustains a severe laceration to her left arm. Amongst

her injuries it is noticed that she has lost the ability to

adduct the fingers of her left hand.

Ulnar nerve

The interossei are supplied by the ulnar nerve.

18. A 28 year old rugby player injures his right humerus and

on examination is noted to have a minor sensory deficit

overlying the point of deltoid insertion into the humerus.

Axillary nerve

This patch of skin is supplied by the axillary nerve

Next question

Brachial plexus

Origin Anterior rami of C5 to T1

Sections of the

plexus

Roots, trunks, divisions, cords, branches

Mnemonic:Real Teenagers Drink Cold Beer

Roots Located in the posterior triangle

Pass between scalenus anterior and medius

Page 7: MRCS preparation emrcs questions upperlimb

Trunks Located posterior to middle third of clavicle

Upper and middle trunks related superiorly to the

subclavian artery

Lower trunk passes over 1st rib posterior to the

subclavian artery

Divisions Apex of axilla

Cords Related to axillary artery

Diagram illustrating the branches of the brachial plexus

Image sourced from Wikipedia

Cutaneous sensation of the upper limb

Page 8: MRCS preparation emrcs questions upperlimb

Image sourced from Wikipedia

A 21 year old man is stabbed in the antecubital fossa. A

decision is made to surgically explore the wound. At

operation the surgeon dissects down onto the brachial

artery. A nerve is identified medially, which nerve is it likely

to be?

A. Radial

B. Recurrent branch of median

C. Anterior interosseous

D. Ulnar

E. Median

Next question

Page 9: MRCS preparation emrcs questions upperlimb

Theme from September 2012 Exam

Median nerve

The median nerve is formed by the union of a lateral and

medial root respectively from the lateral (C5,6,7) and

medial (C8 and T1) cords of the brachial plexus; the

medial root passes anterior to the third part of the axillary

artery. The nerve descends lateral to the brachial artery,

crosses to its medial side (usually passing anterior to the

artery). It passes deep to the bicipital aponeurosis and the

median cubital vein at the elbow.

It passes between the two heads of the pronator teres

muscle, and runs on the deep surface of flexor digitorum

superficialis (within its fascial sheath).

Near the wrist it becomes superficial between the tendons

of flexor digitorum superficialis and flexor carpi radialis,

deep to palmaris longus tendon. It passes deep to the

flexor retinaculum to enter the palm, but lies anterior to the

long flexor tendons within the carpal tunnel.

Branches

Region Branch

Upper arm No branches, although the nerve commonly communicates

with the musculocutaneous nerve

Forearm Pronator teres

Flexor carpi radialis

Page 10: MRCS preparation emrcs questions upperlimb

Palmaris longus

Flexor digitorum superficialis

Flexor pollicis longus

Flexor digitorum profundus (only the radial half)

Distal

forearm

Palmar cutaneous branch

Hand

(Motor)

Motor supply (LOAF)

Lateral 2 lumbricals

Opponens pollicis

Abductor pollicis brevis

Flexor pollicis brevis

Hand

(Sensory)

Over thumb and lateral 2 ½ fingers

On the palmar aspect this projects proximally, on the

dorsal aspect only the distal regions are innervated with

the radial nerve providing the more proximal cutaneous

innervation.

Patterns of damage

Damage at wrist

e.g. carpal tunnel syndrome

paralysis and wasting of thenar eminence muscles

and opponens pollicis (ape hand deformity)

sensory loss to palmar aspect of lateral (radial) 2 ½

fingers

Damage at elbow, as above plus:

unable to pronate forearm

Page 11: MRCS preparation emrcs questions upperlimb

weak wrist flexion

ulnar deviation of wrist

Anterior interosseous nerve (branch of median nerve)

leaves just below the elbow

results in loss of pronation of forearm and weakness

of long flexors of thumb and index finger

Topography of the median nerve

Image sourced fro

A man sustains a laceration between the base of the little

finger and wrist. Several weeks after the injury there is

Page 12: MRCS preparation emrcs questions upperlimb

loss of thumb adduction power. Which nerve is most likely

to have been injured?

A. Superficial ulnar nerve

B. Deep ulnar nerve

C. Median nerve

D. Radial nerve

E. Recurrent branch of median nerve

Next question

Theme from 2009 Exam

Ulnar nerve injury at wrist

Branches of the ulnar nerve in the wrist and hand

At the wrist the ulnar nerve divides into superficial and

deep branches. The superficial branch lies deep to the

palmaris brevis. It divides into two; to produce digital

nerves, which innervate the skin of the medial third of the

palm and the palmar surface of one and a half fingers.

The deep branch arises from the nerve on the flexor

retinaculum lateral to the pisiform bone. It passes

posteriorly between the abductor and short flexor of the

little finger supplying them, and supplying and piercing the

opponens digiti minimi near its origin from the flexor

retinaculum, turns laterally over the distal surface of the

Page 13: MRCS preparation emrcs questions upperlimb

Hook of the Hamate bone. It eventually passes between

the two heads of adductor pollicis with the deep palmar

arch and ends in the first dorsal interosseous muscle. In

the palm the deep branch also innervates the lumbricals

and interosseous muscles.

A 25 year old man is stabbed in the upper arm. The

brachial artery is lacerated at the level of the proximal

humerus, and is being repaired. A nerve lying immediately

lateral to the brachial artery is also lacerated. Which of the

following is the nerve most likely to be?

A. Ulnar nerve

B. Median nerve

C. Radial nerve

D. Intercostobrachial nerve

E. Axillary nerve

Next question

The brachial artery begins at the lower border of teres

major and terminates in the cubital fossa by branching into

the radial and ulnar arteries. In the upper arm the median

nerve lies closest to it in the lateral position. In the cubital

fossa it lies medial to it.

Page 14: MRCS preparation emrcs questions upperlimb

Image sourced from Wikipedia

Brachial artery

The brachial artery begins at the lower border of teres

major as a continuation of the axillary artery. It terminates

in the cubital fossa at the level of the neck of the radius by

dividing into the radial and ulnar arteries.

Relations

Posterior relations include the long head of triceps with the

radial nerve and profunda vessels intervening. Anteriorly it

is overlapped by the medial border of biceps.

It is crossed by the median nerve in the middle of the arm.

In the cubital fossa it is separated from the median cubital

vein by the bicipital aponeurosis.

Page 15: MRCS preparation emrcs questions upperlimb

The basilic vein is in contact at the most proximal aspect

of the cubital fossa and lies medially.

What is the course of the median nerve relative to the

brachial artery in the upper arm?

A. Medial to anterior to lateral

B. Lateral to posterior to medial

C. Medial to posterior to lateral

D. Medial to anterior to medial

E. Lateral to anterior to medial

Next question

Relations of median nerve to the brachial artery:

Lateral -> Anterior -> Medial

Theme from 2009 and 2012 Exams

The median nerve descends lateral to the brachial artery,

it usually passes anterior to the artery to lie on its medial

side. It passes deep to the bicipital aponeurosis and the

median cubital vein at the elbow. It enters the forearm

between the two heads of the pronator teres muscle.

Page 16: MRCS preparation emrcs questions upperlimb

Image sourced from Wikipedia

Brachial artery

The brachial artery begins at the lower border of teres

major as a continuation of the axillary artery. It terminates

in the cubital fossa at the level of the neck of the radius by

dividing into the radial and ulnar arteries.

Relations

Posterior relations include the long head of triceps with the

radial nerve and profunda vessels intervening. Anteriorly it

is overlapped by the medial border of biceps.

It is crossed by the median nerve in the middle of the arm.

In the cubital fossa it is separated from the median cubital

vein by the bicipital aponeurosis.

The basilic vein is in contact at the most proximal aspect

of the cubital fossa and lies medially.

A 22 year old falls over and lands on a shard of glass. It

penetrates the palmar aspect of his hand, immediately

lateral to the pisiform bone. Which of the following

structures is most likely to be injured?

Page 17: MRCS preparation emrcs questions upperlimb

A. Palmar cutaneous branch of the median nerve

B. Lateral tendons of flexor digitorum superficialis

C. Ulnar artery

D. Flexor carpi radialis tendons

E. Lateral tendons of flexor digitorum profundus

Next question

The ulnar nerve and artery are at most immediate risk in

this injury. This is illustrated in the image below:

Page 18: MRCS preparation emrcs questions upperlimb

Image sourced from Wikipedia

Hand

Anatomy of the hand

Bones 8 Carpal bones

5 Metacarpals

14 phalanges

Intrinsic

Muscles

7 Interossei - Supplied by ulnar nerve

3 palmar-adduct fingers

4 dorsal- abduct fingers

Intrinsic

muscles

Lumbricals

Flex MCPJ and extend the IPJ.

Origin deep flexor tendon and insertion dorsal

extensor hood mechanism.

Innervation: 1st and 2nd- median nerve, 3rd and

4th- deep branch of the ulnar nerve.

Thenar

eminence

Abductor pollicis brevis

Opponens pollicis

Flexor pollicis brevis

Hypothenar

eminence

Opponens digiti minimi

Flexor digiti minimi brevis

Abductor digiti minimi

Page 19: MRCS preparation emrcs questions upperlimb

A motorcyclist is involved in a road traffic accident. He

suffers a complex humeral shaft fracture which is plated.

Post operatively he complains of an inability to extend his

fingers. Which of the following structures is most likely to

have been injured?

A. Ulnar nerve

B. Radial nerve

C. Median nerve

D. Axillary nerve

E. None of the above

Next question

Page 20: MRCS preparation emrcs questions upperlimb

Mnemonic for radial nerve muscles: BEST

B rachioradialis

E xtensors

S upinator

T riceps

The radial nerve is responsible for innervation of the

extensor compartment of the forearm.

Radial nerve

Continuation of posterior cord of the brachial plexus (root

values C5 to T1)

Path

In the axilla: lies posterior to the axillary artery on

subscapularis, latissimus dorsi and teres major.

Enters the arm between the brachial artery and the

long head of triceps (medial to humerus).

Spirals around the posterior surface of the humerus

in the groove for the radial nerve.

At the distal third of the lateral border of the humerus

it then pierces the intermuscular septum and

descends in front of the lateral epicondyle.

Page 21: MRCS preparation emrcs questions upperlimb

At the lateral epicondyle it lies deeply between

brachialis and brachioradialis where it then divides

into a superficial and deep terminal branch.

Deep branch crosses the supinator to become the

posterior interosseous nerve.

In the image below the relationships of the radial nerve

can be appreciated

Image sourced from Wikipedia

Regions innervated

Motor (main

nerve)

Triceps

Anconeus

Brachioradialis

Extensor carpi radialis

Motor

(posterior

Extensor carpi ulnaris

Extensor digitorum

Page 22: MRCS preparation emrcs questions upperlimb

interosseous

branch

Extensor indicis

Extensor digiti minimi

Extensor pollicis longus and brevis

Abductor pollicis longus

Sensory The area of skin supplying the proximal

phalanges on the dorsal aspect of the hand

is supplied by the radial nerve (this does

not apply to the little finger and part of the

ring finger)

Muscular innervation and effect of denervation

Anatomical

location

Muscle affected Effect of paralysis

Shoulder Long head of

triceps

Minor effects on

shoulder stability in

abduction

Arm Triceps Loss of elbow extension

Forearm Supinator

Brachioradialis

Extensor carpi

radialis longus and

brevis

Weakening of

supination of prone

hand and elbow flexion

in mid prone position

The cutaneous sensation of the upper limb- illustrating the

contribution of the radial nerve

Page 23: MRCS preparation emrcs questions upperlimb

A 48 year old lady is undergoing an axillary node

clearance for breast cancer. Which of the structures listed

below are most likely to be encountered during the axillary

dissection?

A. Cords of the brachial plexus

B. Thoracodorsal trunk

C. Internal mammary artery

D. Thoracoacromial artery

E. None of the above

Next question

Page 24: MRCS preparation emrcs questions upperlimb

Beware of damaging the thoracodorsal trunk if a latissimus dorsi

flap reconstruction is planned.

Theme from 2009 Exam

The thoracodorsal trunk runs through the nodes in the

axilla. If injured it may compromise the function and blood

supply to latissimus dorsi, which is significant if it is to be

used as a flap for a reconstructive procedure.

Axilla

Boundaries of the axilla

Medially Chest wall and Serratus anterior

Laterally Humeral head

Floor Subscapularis

Anterior aspect Lateral border of Pectoralis major

Fascia Clavipectoral fascia

Content:

Long thoracic nerve

(of Bell)

Derived from C5-C7 and passes behind the

brachial plexus to enter the axilla. It lies on the

medial chest wall and supplies serratus anterior.

Its location puts it at risk during axillary surgery

and damage will lead to winging of the scapula.

Thoracodorsal nerve

and thoracodorsal

Innervate and vascularise latissimus dorsi.

Page 25: MRCS preparation emrcs questions upperlimb

trunk

Axillary vein Lies at the apex of the axilla, it is the continuation

of the basilic vein. Becomes the subclavian vein

at the outer border of the first rib.

Intercostobrachial

nerves

Traverse the axillary lymph nodes and are often

divided during axillary surgery. They provide

cutaneous sensation to the axillary skin.

Lymph nodes The axilla is the main site of lymphatic drainage

for the breast.

53 year old lady is recovering following a difficult

mastectomy and axillary nodal clearance for carcinoma of

the breast. She complains of shoulder pain and on

examination has obvious winging of the scapula. Loss of

innervation to which of the following is the most likely

underlying cause?

A. Latissimus dorsi

B. Serratus anterior

C. Pectoralis minor

D. Pectoralis major

E. Rhomboids

Next question

Theme from April 2012 Exam

Winging of the scapula is most commonly the result of

Page 26: MRCS preparation emrcs questions upperlimb

long thoracic nerve injury or dysfunction. Iatrogenic

damage during the course of the difficult axillary dissection

is the most likely cause in this scenario. Damage to the

rhomboids may produce winging of the scapula but would

be rare in the scenario given.

Long thoracic nerve

Derived from ventral rami of C5, C6, and C7 (close to

their emergence from intervertebral foramina)

It runs downward and passes either anterior or

posterior to the middle scalene muscle

It reaches upper tip of serratus anterior muscle and

descends on outer surface of this muscle, giving

branches into it

Winging of Scapula occurs in long thoracic nerve

injury (most common) or from spinal accessory nerve

injury (which denervates the trapezius) or a dorsal

scapular nerve injury

A 23 year old man falls and slips at a nightclub. A shard of

glass penetrates the skin at the level of the medial

epicondyle, which of the following sequelae is least likely

to occur?

A. Atrophy of the first dorsal interosseous muscle

B. Difficulty in abduction of the the 2nd, 3rd, 4th and

5th fingers

Page 27: MRCS preparation emrcs questions upperlimb

C. Claw like appearance of the hand

D. Loss of sensation on the anterior aspect of the 5th

finger

E. Partial denervation of flexor digitorum profundus

Next question

Injury to the ulnar nerve in the mid to distal forearm will

typically produce a claw hand. This consists of flexion of the 4th

and 5th interphalangeal joints and extension of the

metacarpophalangeal joints. The effects are potentiated when

flexor digitorum profundus is not affected, and the clawing is

more pronounced.More proximally sited ulnar nerve lesions

produce a milder clinical picture owing to the simultaneous

paralysis of flexor digitorum profundus (ulnar half).

This is the 'ulnar paradox', due to the more proximal level

of transection the hand will typically not have a claw like

appearance that may be seen following a more distal

injury. The first dorsal interosseous muscle will be affected

as it is supplied by the ulnar nerve.

Ulnar nerve

Origin

C8, T1

Page 28: MRCS preparation emrcs questions upperlimb

Supplies (no muscles in the upper arm)

Flexor carpi ulnaris

Flexor digitorum profundus

Flexor digiti minimi

Abductor digiti minimi

Opponens digiti minimi

Adductor pollicis

Interossei muscle

Third and fourth lumbricals

Palmaris brevis

Path

Posteromedial aspect of ulna to flexor compartment

of forearm, then along the ulnar. Passes beneath the

flexor carpi ulnaris muscle, then superficially through

the flexor retinaculum into the palm of the hand.

Page 29: MRCS preparation emrcs questions upperlimb

Image sourced from Wikipedia

Branches

Branch Supplies

Articular branch Flexor carpi ulnaris

Medial half of the flexor digitorum

profundus

Palmar cutaneous branch (Arises

near the middle of the forearm)

Skin on the medial part of the palm

Dorsal cutaneous branch Dorsal surface of the medial part of the

hand

Superficial branch Cutaneous fibres to the anterior surfaces

of the medial one and one-half digits

Page 30: MRCS preparation emrcs questions upperlimb

Deep branch Hypothenar muscles

All the interosseous muscles

Third and fourth lumbricals

Adductor pollicis

Medial head of the flexor pollicis brevis

Effects of injury

Damage at the

wrist

Wasting and paralysis of intrinsic hand muscles

(claw hand)

Wasting and paralysis of hypothenar muscles

Loss of sensation medial 1 and half fingers

Damage at the

elbow

Radial deviation of the wrist

Clawing less in 3rd and 4th digits

A 43 year old lady is due to undergo an axillary node

clearance as part of treatment for carcinoma of the breast.

Which of the following fascial layers will be divided during

the surgical approach to the axilla?

A. Sibsons fascia

B. Pre tracheal fascia

C. Waldayers fascia

D. Clavipectoral fascia

E. None of the above

Next question

Page 31: MRCS preparation emrcs questions upperlimb

The clavipectoral fascia is situated under the clavicular

portion of pectoralis major. It protects both the axillary

vessels and nodes. During an axillary node clearance for

breast cancer the clavipectoral fascia is incised and this

allows access to the nodal stations. The nodal stations

are; level 1 nodes inferior to pectoralis minor, level 2 lie

behind it and level 3 above it. During a Patey Mastectomy

surgeons divide pectoralis minor to gain access to level 3

nodes. The use of sentinel node biopsy (and stronger

assistants!) have made this procedure far less common.

A 23 year old climber falls and fractures his humerus. The

surgeons decide upon a posterior approach to the middle

third of the bone. Which of the following nerves is at

greatest risk in this approach?

A. Ulnar

B. Antebrachial

C. Musculocutaneous

D. Radial

E. Intercostobrachial

Next question

Theme from April 2012 Exam

The radial nerve wraps around the humerus and may be

Page 32: MRCS preparation emrcs questions upperlimb

injured during a posterior approach. An IM nail may be

preferred as it avoids the complex dissection needed for

direct bone exposure.

Radial nerve

Continuation of posterior cord of the brachial plexus (root

values C5 to T1)

Path

In the axilla: lies posterior to the axillary artery on

subscapularis, latissimus dorsi and teres major.

Enters the arm between the brachial artery and the

long head of triceps (medial to humerus).

Spirals around the posterior surface of the humerus

in the groove for the radial nerve.

At the distal third of the lateral border of the humerus

it then pierces the intermuscular septum and

descends in front of the lateral epicondyle.

At the lateral epicondyle it lies deeply between

brachialis and brachioradialis where it then divides

into a superficial and deep terminal branch.

Deep branch crosses the supinator to become the

posterior interosseous nerve.

In the image below the relationships of the radial nerve

can be appreciated

Page 33: MRCS preparation emrcs questions upperlimb

Image sourced from Wikipedia

Regions innervated

Motor (main

nerve)

Triceps

Anconeus

Brachioradialis

Extensor carpi radialis

Motor

(posterior

interosseous

branch

Extensor carpi ulnaris

Extensor digitorum

Extensor indicis

Extensor digiti minimi

Extensor pollicis longus and brevis

Abductor pollicis longus

Sensory The area of skin supplying the proximal

phalanges on the dorsal aspect of the hand

is supplied by the radial nerve (this does

not apply to the little finger and part of the

ring finger)

Page 34: MRCS preparation emrcs questions upperlimb

Muscular innervation and effect of denervation

Anatomical

location

Muscle affected Effect of paralysis

Shoulder Long head of

triceps

Minor effects on

shoulder stability in

abduction

Arm Triceps Loss of elbow extension

Forearm Supinator

Brachioradialis

Extensor carpi

radialis longus and

brevis

Weakening of

supination of prone

hand and elbow flexion

in mid prone position

The cutaneous sensation of the upper limb- illustrating the

contribution of the radial nerve

Image sourced from Wikipedia

Page 35: MRCS preparation emrcs questions upperlimb

heme: Nerve injury

A. Median nerve

B. Ulnar nerve

C. Radial nerve

D. Posterior interosseous nerve

E. Anterior interosseous nerve

F. Musculocutaneous nerve

G. Axillary nerve

H. Brachial Trunks C5-6

I. Brachial trunks C6-7

J. Brachial Trunks C8-T1

Please select the most likely lesion site for each scenario.

Each option may be used once, more than once or not at

all.

83. A 42 year old teacher is admitted with a fall. An x-ray

confirms a fracture of the surgical neck of the humerus.

Which nerve is at risk?

Axillary nerve

The Axillary nerve winds around the bone at the neck of

the humerus. The axillary nerve is also at risk during

shoulder dislocation.

Page 36: MRCS preparation emrcs questions upperlimb

84. A 32 year old window cleaner is admitted after falling off

the roof. He reports that he had slipped off the top of the

roof and was able to cling onto the gutter for a few

seconds. The patient has Horner's syndrome.

Brachial Trunks C8-T1

The patient has a Klumpke's paralysis involving brachial

trunks C8-T1. Classically there is weakness of the hand

intrinsic muscles. Involvement of T1 may cause a

Horner's syndrome. It occurs as a result of traction

injuries or during delivery.

85. A 32 year old rugby player is hit hard on the shoulder

during a rough tackle. Clinically his arm is hanging loose

on the side. It is pronated and medially rotated.

Brachial Trunks C5-6

The patient has an Erb's palsy involving brachial trunks

C5-6.

Next question

Brachial plexus

Origin Anterior rami of C5 to T1

Sections of the Roots, trunks, divisions, cords, branches

Page 37: MRCS preparation emrcs questions upperlimb

plexus Mnemonic:Real Teenagers Drink Cold Beer

Roots Located in the posterior triangle

Pass between scalenus anterior and medius

Trunks Located posterior to middle third of clavicle

Upper and middle trunks related superiorly to the

subclavian artery

Lower trunk passes over 1st rib posterior to the

subclavian artery

Divisions Apex of axilla

Cords Related to axillary artery

Diagram illustrating the branches of the brachial plexus

Image sourced from Wikipedia

Cutaneous sensation of the upper limb

Page 38: MRCS preparation emrcs questions upperlimb

Image sourced

hich of the following nerves is responsible for innervation

of the triceps muscle?

A. Radial

B. Ulnar

C. Axillary

D. Median

E. None of the above

Next question

Page 39: MRCS preparation emrcs questions upperlimb

To remember nerve roots and their relexes:

1-2 Ankle (S1-S2)

3-4 Knee (L3-L4)

5-6 Biceps (C5-C6)

7-8 Triceps (C7-C8)

The radial nerve innervates all three heads of triceps, with

a separate branch to each head.

Triceps

Origin Long head- infraglenoid tubercle of the scapula.

Lateral head- dorsal surface of the humerus, lateral

and proximal to the groove of the radial nerve

Medial head- posterior surface of the humerus on the

inferomedial side of the radial groove and both of the

intermuscular septae

Insertion Olecranon process of the ulna. Here the olecranon

bursa is between the triceps tendon and olecranon.

Some fibres insert to the deep fascia of the forearm,

posterior capsule of the elbow (preventing the capsule

from being trapped between olecranon and olecranon

fossa during extension)

Innervation Radial nerve

Blood

supply

Profunda brachii artery

Page 40: MRCS preparation emrcs questions upperlimb

Action Elbow extension. The long head can adduct the humerus and

and extend it from a flexed position

Relations The radial nerve and profunda brachii vessels lie between the

lateral and medial heads

Which of the following muscles inserts onto the lesser

tuberostiy of the the humerus?

A. Subscapularis

B. Deltoid

C. Supraspinatus

D. Teres minor

E. Infraspinatus

Next question

With the exception of subscapularis which inserts into the

lesser tuberosity, the muscles of the rotator cuff insert into

the greater tuberosity.

Shoulder joint

Shallow synovial ball and socket type of joint.

It is an inherently unstable joint, but is capable to a

wide range of movement.

Stability is provided by muscles of the rotator cuff that

pass from the scapula to insert in the greater

Page 41: MRCS preparation emrcs questions upperlimb

tuberosity (all except sub scapularis-lesser

tuberosity).

Glenoid labrum

Fibrocartilaginous rim attached to the free edge of

the glenoid cavity

Tendon of the long head of biceps arises from within

the joint from the supraglenoid tubercle, and is fused

at this point to the labrum.

The long head of triceps attaches to the infraglenoid

tubercle

Fibrous capsule

Attaches to the scapula external to the glenoid

labrum and to the labrum itself (postero-superiorly)

Attaches to the humerus at the level of the

anatomical neck superiorly and the surgical neck

inferiorly

Anteriorly the capsule is in contact with the tendon of

subscapularis, superiorly with the supraspinatus

tendon, and posteriorly with the tendons of

infraspinatus and teres minor. All these blend with

the capsule towards their insertion.

Two defects in the fibrous capsule; superiorly for the

tendon of biceps. Anteriorly there is a defect beneath

the subscapularis tendon.

Page 42: MRCS preparation emrcs questions upperlimb

The inferior extension of the capsule is closely

related to the axillary nerve at the surgical neck and

this nerve is at risk in anteroinferior dislocations. It

also means that proximally sited osteomyelitis may

progress to septic arthritis.

Movements and muscles

Flexion Anterior part of deltoid

Pectoralis major

Biceps

Coracobrachialis

Extension Posterior deltoid

Teres major

Latissimus dorsi

Adduction Pectoralis major

Latissimus dorsi

Teres major

Coracobrachialis

Abduction Mid deltoid

Supraspinatus

Medial rotation Subscapularis

Anterior deltoid

Teres major

Latissimus dorsi

Lateral rotation Posterior deltoid

Infraspinatus

Teres minor

Important anatomical relations

Page 43: MRCS preparation emrcs questions upperlimb

Anteriorly Brachial plexus

Axillary artery and vein

Posterior Suprascapular nerve

Suprascapular vessels

Inferior Axillary nerve

Circumflex humeral vessels

hich of the following nerves is not contained within the

posterior triangle of the neck?

A. Accessory nerve

B. Phrenic nerve

C. Greater auricular nerve

D. Ansa cervicalis

E. Lesser occiptal nerve

Next question

Theme from September 2012 Exam

Ansa cervicalis is a content of the anterior triangle of the

neck.

Posterior triangle of the neck

Boundaries

Apex Sternocleidomastoid and the Trapezius muscles at the Occipital

bone

Page 44: MRCS preparation emrcs questions upperlimb

Anterior Posterior border of the Sternocleidomastoid

Posterior Anterior border of the Trapezius

Base Middle third of the clavicle

Image sourced from Wikipedia

Contents

Nerves Accessory nerve

Phrenic nerve

Three trunks of the brachial plexus

Branches of the cervical plexus: Supraclavicular nerve,

transverse cervical nerve, great auricular nerve, lesser

occipital nerve

Vessels External jugular vein

Subclavian artery

Muscles Inferior belly of omohyoid

Scalene

Lymph

nodes

Supraclavicular

Page 45: MRCS preparation emrcs questions upperlimb

Occipital

A 73 year old lady suffers a fracture at the surgical neck of

the humerus. The decision is made to operate. There are

difficulties in reducing the fracture and a vessel lying

posterior to the surgical neck is injured. Which of the

following is this vessel most likely to be?

A. Axillary artery

B. Brachial artery

C. Thoracoacromial artery

D. Transverse scapular artery

E. Posterior circumflex humeral artery

Next question

The circumflex humeral arteries lie at the surgical neck

and is this scenario the posterior circumflex is likely to be

injured. The thoracoacromial and transverse scapular

arteries lie more superomedially. The posterior circumflex

humeral artery is a branch of the axillary artery.

Shoulder joint

Shallow synovial ball and socket type of joint.

Page 46: MRCS preparation emrcs questions upperlimb

It is an inherently unstable joint, but is capable to a

wide range of movement.

Stability is provided by muscles of the rotator cuff that

pass from the scapula to insert in the greater

tuberosity (all except sub scapularis-lesser

tuberosity).

Glenoid labrum

Fibrocartilaginous rim attached to the free edge of

the glenoid cavity

Tendon of the long head of biceps arises from within

the joint from the supraglenoid tubercle, and is fused

at this point to the labrum.

The long head of triceps attaches to the infraglenoid

tubercle

Fibrous capsule

Attaches to the scapula external to the glenoid

labrum and to the labrum itself (postero-superiorly)

Attaches to the humerus at the level of the

anatomical neck superiorly and the surgical neck

inferiorly

Anteriorly the capsule is in contact with the tendon of

subscapularis, superiorly with the supraspinatus

tendon, and posteriorly with the tendons of

Page 47: MRCS preparation emrcs questions upperlimb

infraspinatus and teres minor. All these blend with

the capsule towards their insertion.

Two defects in the fibrous capsule; superiorly for the

tendon of biceps. Anteriorly there is a defect beneath

the subscapularis tendon.

The inferior extension of the capsule is closely

related to the axillary nerve at the surgical neck and

this nerve is at risk in anteroinferior dislocations. It

also means that proximally sited osteomyelitis may

progress to septic arthritis.

Movements and muscles

Flexion Anterior part of deltoid

Pectoralis major

Biceps

Coracobrachialis

Extension Posterior deltoid

Teres major

Latissimus dorsi

Adduction Pectoralis major

Latissimus dorsi

Teres major

Coracobrachialis

Abduction Mid deltoid

Supraspinatus

Medial rotation Subscapularis

Anterior deltoid

Teres major

Latissimus dorsi

Page 48: MRCS preparation emrcs questions upperlimb

Lateral rotation Posterior deltoid

Infraspinatus

Teres minor

Important anatomical relations

Anteriorly Brachial plexus

Axillary artery and vein

Posterior Suprascapular nerve

Suprascapular vessels

Inferior Axillary nerve

Circumflex humeral vessels

Which of the structures listed below lies posterior to the

carotid sheath at the level of the 6th cervical vertebra?

A. Hypoglossal nerve

B. Vagus nerve

C. Cervical sympathetic chain

D. Ansa cervicalis

E. Glossopharyngeal nerve

Next question

The carotid sheath is crossed anteriorly by the

hypoglossal nerves and the ansa cervicalis. The vagus

lies within it. The cervical sympathetic chain lies

Page 49: MRCS preparation emrcs questions upperlimb

posteriorly between the sheath and the prevertebral

fascia.

Common carotid artery

The right common carotid artery arises at the bifurcation of

the brachiocephalic trunk, the left common carotid arises

from the arch of the aorta. Both terminate at the level of

the upper border of the thyroid cartilage (the lower border

of the third cervical vertebra) by dividing into the internal

and external carotid arteries.

Left common carotid artery

This vessel arises immediately to the left and slightly

behind the origin of the brachiocephalic trunk. Its thoracic

portion is 2.5- 3.5 cm in length and runs superolaterally to

the sternoclavicular joint.

In the thorax

The vessel is in contact, from below upwards, with the

trachea, left recurrent laryngeal nerve, left margin of the

oesophagus. Anteriorly the left brachiocephalic vein runs

across the artery, and the cardiac branches from the left

vagus descend in front of it. These structures together

with the thymus and the anterior margins of the left lung

and pleura separate the artery from the manubrium.

In the neck

The artery runs superiorly deep to sternocleidomastoid

Page 50: MRCS preparation emrcs questions upperlimb

and then enters the anterior triangle. At this point it lies

within the carotid sheath with the vagus nerve and the

internal jugular vein. Posteriorly the sympathetic trunk lies

between the vessel and the prevertebral fascia. At the

level of C7 the vertebral artery and thoracic duct lie behind

it. The anterior tubercle of C6 transverse process is

prominent and the artery can be compressed against this

structure (it corresponds to the level of the cricoid).

Anteriorly at C6 the omohyoid muscle passes superficial

to the artery.

Within the carotid sheath the jugular vein lies lateral to the

artery.

Right common carotid artery

The right common carotid arises from the brachiocephalic

artery. The right common carotid artery corresponds with

the cervical portion of the left common carotid, except that

there is no thoracic duct on the right. The oesophagus is

less closely related to the right carotid than the left.

Summary points about the carotid anatomy

Path

Passes behind the sternoclavicular joint (12% patients

above this level) to the upper border of the thyroid

cartilage, to divide into the external (ECA) and internal

carotid arteries (ICA).

Relations

Page 51: MRCS preparation emrcs questions upperlimb

Level of 6th cervical vertebra crossed by omohyoid

Then passes deep to the thyrohyoid, sternohyoid,

sternomastoid muscles.

Passes behind the carotid tubercle (transverse

process 6th cervical vertebra)-NB compression here

stops haemorrhage.

The inferior thyroid artery passes posterior to the

common carotid artery.

Then : Left common carotid artery crossed by

thoracic duct, Right common carotid artery crossed

by recurrent laryngeal nerve

Image sourced from Wikipedia

A 45 year old man presents with a lipoma located

posterior to the posterior border of the

Page 52: MRCS preparation emrcs questions upperlimb

sternocleidomastoid muscle, approximately 4cm superior

to the middle third of the clavicle. During surgical excision

of the lesion troublesome bleeding is encountered. Which

of the following is the most likely source?

A. Internal jugular vein

B. External jugular vein

C. Common carotid artery

D. Vertebral artery

E. Second part of the subclavian artery

Next question

The external jugular vein runs obliquely in the superficial

fascia of the posterior triangle. It drains into the subclavian

vein. During surgical exploration of this area the external

jugular vein may be injured and troublesome bleeding may

result. The internal jugular vein and carotid arteries are

located in the anterior triangle. The third, and not the

second, part of the subclavian artery is also a content of

the posterior triangle

Posterior triangle of the neck

Boundaries

Apex Sternocleidomastoid and the Trapezius muscles at the Occipital

bone

Page 53: MRCS preparation emrcs questions upperlimb

Anterior Posterior border of the Sternocleidomastoid

Posterior Anterior border of the Trapezius

Base Middle third of the clavicle

Image sourced from Wikipedia

Contents

Nerves Accessory nerve

Phrenic nerve

Three trunks of the brachial plexus

Branches of the cervical plexus: Supraclavicular nerve,

transverse cervical nerve, great auricular nerve, lesser

occipital nerve

Vessels External jugular vein

Subclavian artery

Muscles Inferior belly of omohyoid

Scalene

Lymph

nodes

Supraclavicular

Page 54: MRCS preparation emrcs questions upperlimb

Occipital

Which of the following upper limb muscles is not innervated by the

radial nerve?

A. Extensor carpi ulnaris

B. Abductor digit minimi

C. Anconeus

D. Supinator

E. Brachioradialis

Next question

Mnemonic for radial nerve muscles: BEST

B rachioradialis

E xtensors

S upinator

T riceps

Abductor digiti minimi is innervated by the ulnar nerve.

Radial nerve

Continuation of posterior cord of the brachial plexus (root values C5

to T1)

Path

In the axilla: lies posterior to the axillary artery on

subscapularis, latissimus dorsi and teres major.

Page 55: MRCS preparation emrcs questions upperlimb

Enters the arm between the brachial artery and the long head

of triceps (medial to humerus).

Spirals around the posterior surface of the humerus in the

groove for the radial nerve.

At the distal third of the lateral border of the humerus it then

pierces the intermuscular septum and descends in front of the

lateral epicondyle.

At the lateral epicondyle it lies deeply between brachialis and

brachioradialis where it then divides into a superficial and deep

terminal branch.

Deep branch crosses the supinator to become the posterior

interosseous nerve.

In the image below the relationships of the radial nerve can be

appreciated

Image sourced from Wikipedia

Regions innervated

Motor (main

nerve)

Triceps

Anconeus

Brachioradialis

Page 56: MRCS preparation emrcs questions upperlimb

Extensor carpi radialis

Motor (posterior

interosseous

branch

Extensor carpi ulnaris

Extensor digitorum

Extensor indicis

Extensor digiti minimi

Extensor pollicis longus and brevis

Abductor pollicis longus

Sensory The area of skin supplying the proximal phalanges on

the dorsal aspect of the hand is supplied by the radial

nerve (this does not apply to the little finger and part

of the ring finger)

Muscular innervation and effect of denervation

Anatomical

location

Muscle affected Effect of paralysis

Shoulder Long head of triceps Minor effects on shoulder

stability in abduction

Arm Triceps Loss of elbow extension

Forearm Supinator

Brachioradialis

Extensor carpi radialis

longus and brevis

Weakening of supination of

prone hand and elbow flexion in

mid prone position

The cutaneous sensation of the upper limb- illustrating the

contribution of the radial nerve

Page 57: MRCS preparation emrcs questions upperlimb

Which of the following forms the floor of the anatomical snuffbox?

A. Radial artery

B. Cephalic vein

C. Extensor pollicis brevis

D. Scaphoid bone

E. Cutaneous branch of the radial nerve

Next question

The scaphoid bone forms the floor of the anatomical snuffbox. The

cutaneous branch of the radial nerve is much more superficially and

proximally located.

Anatomical snuffbox

Posterior border Tendon of extensor pollicis longus

Page 58: MRCS preparation emrcs questions upperlimb

Anterior border Tendons of extensor pollicis brevis and abductor pollicis longus

Proximal border Styloid process of the radius

Distal border Apex of snuffbox triangle

Floor Trapezium and scaphoid

Content Radial artery

Image showing the anatomical snuffbox

A 32 year old lady complains of carpal tunnel syndrome. The carpal

tunnel is explored surgically. Which of the following structures will lie

in closest proximity to the hamate bone within the carpal tunnel?

A. The tendon of abductor pollicis longus

B. The tendons of flexor digitorum profundus

C. The tendons of flexor carpi radialis longus

D. Median nerve

E. Radial artery

Next question

The carpal tunnel contains nine flexor tendons:

Flexor digitorum profundus

Page 59: MRCS preparation emrcs questions upperlimb

Flexor digitorum superficialis

Flexor pollicis longus

The tendon of flexor digitorum profundus lies deepest in the tunnel

and will thus lie nearest to the hamate bone.

Carpal bones

Diagrammatic image of carpal bones

Image sourced from Wikipedia

Key to image

A Scaphoid

B Lunate

C Triquetrum

D Pisiform

E Trapezium

F Trapezoid

G Capitate

H Hamate

Page 60: MRCS preparation emrcs questions upperlimb

1 Radius

2 Ulna

3 Metacarpals

No tendons attach to: Scaphoid, lunate, triquetrum (stabilised

by ligaments)

A 45 year man presents with hand weakness. He is given a piece of

paper to hold between his thumb and index finger. When the paper is

pulled, the patient has difficulty maintaining a grip. Grip pressure is

maintained by flexing the thumb at the interphalangeal joint. What is

the most likely nerve lesion?

A. Posterior interosseous nerve

B. Deep branch of ulnar nerve

C. Anterior interosseous nerve

D. Superficial branch of the ulnar nerve

E. Radial nerve

Next question

Theme from January 2012 exam

This is a description of Froment's sign, which tests for ulnar nerve

palsy. It mainly tests for the function of adductor pollicis. This is

supplied by the deep branch of the ulnar nerve. Remember the

anterior interosseous branch, which innervates the flexor pollicis

longus (hence causing flexion of the thumb IP joint), branches off

more proximally to the wrist.

Ulnar nerve

Page 61: MRCS preparation emrcs questions upperlimb

Origin

C8, T1

Supplies (no muscles in the upper arm)

Flexor carpi ulnaris

Flexor digitorum profundus

Flexor digiti minimi

Abductor digiti minimi

Opponens digiti minimi

Adductor pollicis

Interossei muscle

Third and fourth lumbricals

Palmaris brevis

Path

Posteromedial aspect of ulna to flexor compartment of forearm,

then along the ulnar. Passes beneath the flexor carpi ulnaris

muscle, then superficially through the flexor retinaculum into

the palm of the hand.

Page 62: MRCS preparation emrcs questions upperlimb

Image sourced from Wikipedia

Branches

Branch Supplies

Articular branch Flexor carpi ulnaris

Medial half of the flexor digitorum profundus

Palmar cutaneous branch (Arises near the

middle of the forearm)

Skin on the medial part of the palm

Dorsal cutaneous branch Dorsal surface of the medial part of the hand

Superficial branch Cutaneous fibres to the anterior surfaces of the

medial one and one-half digits

Deep branch Hypothenar muscles

All the interosseous muscles

Third and fourth lumbricals

Adductor pollicis

Medial head of the flexor pollicis brevis

Page 63: MRCS preparation emrcs questions upperlimb

Effects of injury

Damage at the wrist Wasting and paralysis of intrinsic hand muscles (claw hand)

Wasting and paralysis of hypothenar muscles

Loss of sensation medial 1 and half fingers

Damage at the

elbow

Radial deviation of the wrist

Clawing less in 3rd and 4th digits

A 10 year old by falls out of a tree has suffers a supracondylar

fracture. He complains of a painful elbow and forearm. There is an

obvious loss of pincer movement involving the thumb and index

finger with minimal loss of sensation. The most likely nerve injury is

to the:

A. Ulnar nerve

B. Radial nerve

C. Anterior interosseous nerve

D. Axillary nerve damage

E. Median nerve damage above the elbow

Next question

The anterior interosseous nerve is a motor branch of the median

nerve just below the elbow. When damaged it classically causes:

Pain in the forearm

Loss of pincer movement of the thumb and index finger

(innervates the long flexor muscles of flexor pollicis longus &

flexor digitorum profundus of the index and middle finger)

Minimal loss of sensation due to lack of a cutaneous branch

Page 64: MRCS preparation emrcs questions upperlimb

Median nerve

The median nerve is formed by the union of a lateral and medial root

respectively from the lateral (C5,6,7) and medial (C8 and T1) cords of

the brachial plexus; the medial root passes anterior to the third part of

the axillary artery. The nerve descends lateral to the brachial artery,

crosses to its medial side (usually passing anterior to the artery). It

passes deep to the bicipital aponeurosis and the median cubital vein

at the elbow.

It passes between the two heads of the pronator teres muscle, and

runs on the deep surface of flexor digitorum superficialis (within its

fascial sheath).

Near the wrist it becomes superficial between the tendons of flexor

digitorum superficialis and flexor carpi radialis, deep to palmaris

longus tendon. It passes deep to the flexor retinaculum to enter the

palm, but lies anterior to the long flexor tendons within the carpal

tunnel.

Branches

Region Branch

Upper arm No branches, although the nerve commonly communicates with the

musculocutaneous nerve

Forearm Pronator teres

Flexor carpi radialis

Palmaris longus

Flexor digitorum superficialis

Flexor pollicis longus

Flexor digitorum profundus (only the radial half)

Distal

forearm

Palmar cutaneous branch

Hand

(Motor)

Motor supply (LOAF)

Lateral 2 lumbricals

Opponens pollicis

Abductor pollicis brevis

Page 65: MRCS preparation emrcs questions upperlimb

Flexor pollicis brevis

Hand

(Sensory)

Over thumb and lateral 2 ½ fingers

On the palmar aspect this projects proximally, on the dorsal aspect only

the distal regions are innervated with the radial nerve providing the

more proximal cutaneous innervation.

Patterns of damage

Damage at wrist

e.g. carpal tunnel syndrome

paralysis and wasting of thenar eminence muscles and

opponens pollicis (ape hand deformity)

sensory loss to palmar aspect of lateral (radial) 2 ½ fingers

Damage at elbow, as above plus:

unable to pronate forearm

weak wrist flexion

ulnar deviation of wrist

Anterior interosseous nerve (branch of median nerve)

leaves just below the elbow

results in loss of pronation of forearm and weakness of long

flexors of thumb and index finger

Topography of the median nerve

Page 66: MRCS preparation emrcs questions upperlimb

Image sourced from Wikipedia

A 32 year old attends neurology clinic complaining of tingling in his

hand. He has radial deviation of his wrist and there is mild clawing of

his fingers, with the 3rd and 4th digits being relatively spared. What is

the most likely lesion?

A. Ulnar nerve damage at the wrist

B. Ulnar nerve damage at the elbow

C. Radial nerve damage at the elbow

D. Median nerve damage at the wrist

E. Median nerve damage at the elbow

Next question

Page 67: MRCS preparation emrcs questions upperlimb

At the elbow the ulnar nerve lesion affects the flexor carpi ulnaris and

flexor digitorum profundus.

Ulnar nerve

Origin

C8, T1

Supplies (no muscles in the upper arm)

Flexor carpi ulnaris

Flexor digitorum profundus

Flexor digiti minimi

Abductor digiti minimi

Opponens digiti minimi

Adductor pollicis

Interossei muscle

Third and fourth lumbricals

Palmaris brevis

Path

Posteromedial aspect of ulna to flexor compartment of forearm,

then along the ulnar. Passes beneath the flexor carpi ulnaris

muscle, then superficially through the flexor retinaculum into

the palm of the hand.

Page 68: MRCS preparation emrcs questions upperlimb

Image sourced from Wikipedia

Branches

Branch Supplies

Articular branch Flexor carpi ulnaris

Medial half of the flexor digitorum profundus

Palmar cutaneous branch (Arises near the

middle of the forearm)

Skin on the medial part of the palm

Dorsal cutaneous branch Dorsal surface of the medial part of the hand

Superficial branch Cutaneous fibres to the anterior surfaces of the

medial one and one-half digits

Deep branch Hypothenar muscles

All the interosseous muscles

Third and fourth lumbricals

Adductor pollicis

Medial head of the flexor pollicis brevis

Page 69: MRCS preparation emrcs questions upperlimb

Effects of injury

Damage at the wrist Wasting and paralysis of intrinsic hand muscles (claw hand)

Wasting and paralysis of hypothenar muscles

Loss of sensation medial 1 and half fingers

Damage at the

elbow

Radial deviation of the wrist

Clawing less in 3rd and 4th digits

A 23 year old man is involved in a fight and is stabbed in his upper

arm. The ulnar nerve is transected. Which of the following muscles

will not demonstrate compromised function as a result?

A. Flexor carpi ulnaris

B. Medial half of flexor digitorum profundus

C. Palmaris brevis

D. Hypothenar muscles

E. Pronator teres

Next question

M edial lumbricals

A dductor pollicis

F lexor digitorum profundus/Flexor digiti minimi

I nterossei

A bductor digiti minimi and opponens

Innervates all intrinsic muscles of the hand (EXCEPT 2: thenar muscles & first

two lumbricals - supplied by median nerve)

Page 70: MRCS preparation emrcs questions upperlimb

Pronator teres is innervated by the median nerve. Palmaris brevis is

innervated by the ulnar nerve

Ulnar nerve

Origin

C8, T1

Supplies (no muscles in the upper arm)

Flexor carpi ulnaris

Flexor digitorum profundus

Flexor digiti minimi

Abductor digiti minimi

Opponens digiti minimi

Adductor pollicis

Interossei muscle

Third and fourth lumbricals

Palmaris brevis

Path

Posteromedial aspect of ulna to flexor compartment of forearm,

then along the ulnar. Passes beneath the flexor carpi ulnaris

muscle, then superficially through the flexor retinaculum into

the palm of the hand.

Page 71: MRCS preparation emrcs questions upperlimb

Image sourced from Wikipedia

Branches

Branch Supplies

Articular branch Flexor carpi ulnaris

Medial half of the flexor digitorum profundus

Palmar cutaneous branch (Arises near the

middle of the forearm)

Skin on the medial part of the palm

Dorsal cutaneous branch Dorsal surface of the medial part of the hand

Superficial branch Cutaneous fibres to the anterior surfaces of the

medial one and one-half digits

Deep branch Hypothenar muscles

All the interosseous muscles

Third and fourth lumbricals

Adductor pollicis

Medial head of the flexor pollicis brevis

Page 72: MRCS preparation emrcs questions upperlimb

Effects of injury

Damage at the wrist Wasting and paralysis of intrinsic hand muscles (claw hand)

Wasting and paralysis of hypothenar muscles

Loss of sensation medial 1 and half fingers

Damage at the

elbow

Radial deviation of the wrist

Clawing less in 3rd and 4th digits

Which of the structures listed below overlies the cephalic vein?

A. Extensor retinaculum

B. Bicipital aponeurosis

C. Biceps muscle

D. Antebrachial fascia

E. None of the above

Next question

The cephalic vein is superficially located in the upper limb and

overlies most the fascial planes. It pierces the coracoid membrane

(continuation of the clavipectoral fascia) to terminate in the axillary

vein. It lies anterolaterally to biceps.

Cephalic vein

Path

Dorsal venous arch drains laterally into the cephalic vein

Crosses the anatomical snuffbox and travels laterally up the

arm

At the antecubital fossa connected to the basilic vein by the

median cubital vein

Page 73: MRCS preparation emrcs questions upperlimb

Pierces deep fascia of deltopectoral groove to join axillary vein

A 22 year old man is involved in a fight. He sustains a laceration to

the posterior aspect of his wrist. In the emergency department the

wound is explored and the laceration is found to be transversely

orientated and overlies the region of the extensor retinaculum, which

is intact. Which of the following structures is least likely to be injured

in this scenario?

A. Dorsal cutaneous branch of the ulnar nerve

B. Tendon of extensor indicis

C. Basilic vein

D. Superficial branch of the radial nerve

E. Cephalic vein

Next question

The extensor retinaculum attaches to the radius proximal to the

styloid, thereafter it runs obliquely and distally to wind around the

ulnar styloid (but does not attach to it). The extensor tendons lie deep

to the extensor retinaculum and would therefore be less susceptible

to injury than the superficial structures.

Extensor retinaculum

The extensor rentinaculum is a thickening of the deep fascia that

stretches across the back of the wrist and holds the long extensor

tendons in position.

Its attachments are:

The pisiform and hook of hamate medially

The end of the radius laterally

Page 74: MRCS preparation emrcs questions upperlimb

Structures related to the extensor retinaculum

Structures superficial to the

retinaculum

Basilic vein

Dorsal cutaneous branch of the ulnar nerve

Cephalic vein

Superficial branch of the radial nerve

Structures passing deep to the

extensor retinaculum

Extensor carpi ulnaris tendon

Extensor digiti minimi tendon

Extensor digitorum and extensor indicis

tendon

Extensor pollicis longus tendon

Extensor carpi radialis longus tendon

Abductor pollicis longus and extensor

pollicis brevis tendons

Beneath the extensor retinaculum fibrous septa form six

compartments that contain the extensor muscle tendons. Each

compartment has its own synovial sheath.

The radial artery

The radial artery passes between the lateral collateral ligament of the

wrist joint and the tendons of the abductor pollicis longus and

extensor pollicis brevis.

Image illustrating the topography of tendons passing under the

extensor retinaculum

Page 75: MRCS preparation emrcs questions upperlimb

A man has an incision sited than runs 8cm from the deltopectoral

groove to the midline. Which of the following is not at risk of injury?

A. Cephalic vein

B. Shoulder joint capsule

C. Axillary artery

D. Pectoralis major

E. Trunk of the brachial plexus

Next question

Theme from April 2012 Exam

This region will typically lie medial to the joint capsule. The diagram

below illustrates the plane that this would transect and as it can be

appreciated the other structures are all at risk of injury.

Page 76: MRCS preparation emrcs questions upperlimb

Image sourced from Wikipedia

Pectoralis major muscle

Origin From the medial two thirds of the clavicle, manubrium and sternocostal angle

Insertion Crest of the greater tubercle of the humerus

Nerve supply Lateral pectoral nerve

Actions Adductor and medial rotator of the humerus

heme: Nerve Injury

Page 77: MRCS preparation emrcs questions upperlimb

A. Median nerve

B. Ulnar nerve

C. Radial nerve

D. Musculocutaneous nerve

E. Axillary nerve

F. Anterior interosseous nerve

G. Posterior interosseous nerve

For each scenario please select the most likely underlying nerve injury. Each option may be used once, more than once or not at all.

171. A 19 year old student is admitted to A&E after falling off a

wall. He is unable to flex his index finger. An x-ray confirms a

supracondylar fracture.

You answered Anterior interosseous nerve

The correct answer is Median nerve

This median nerve is at risk during a supracondylar fracture.

172. A well toned weight lifter attends clinic reporting weakness of

his left arm. There is weakness of flexion and supination of

the forearm.

You answered Posterior interosseous nerve

The correct answer is Musculocutaneous nerve

Mucocutaneous nerve compression due to entrapment of the

nerve between biceps and brachialis. Elbow flexion and

supination of the arm are affected. This is a rare isolated

injury.

Page 78: MRCS preparation emrcs questions upperlimb

173. An 18 year old girl sustains an Holstein-Lewis fracture. Which

nerve is at risk?

You answered Axillary nerve

The correct answer is Radial nerve

Proximal lesions affect the triceps. Also paralysis of wrist

extensors and forearm supinators occur. Reduced sensation

of dorsoradial aspect of hand and dorsal 31/2 fingers.

Holstein-Lewis fractures are fractures of the distal humerus

with radial nerve entrapment.

A 35 year old farm labourer is injures the posterior aspect of his hand with a

mechanical scythe. He severs some of his extensor tendons in this injury. How many

tunnels lie in the extensor retinaculum that transmit the tendons of the extensor

muscles?

A. One

B. Three

C. Four

D. Five

E. Six

Next question

There are six tunnels, each lined by its own synovial sheath.

Extensor retinaculum

The extensor rentinaculum is a thickening of the deep fascia that stretches across

the back of the wrist and holds the long extensor tendons in position.

Its attachments are:

The pisiform and hook of hamate medially

The end of the radius laterally

Page 79: MRCS preparation emrcs questions upperlimb

Structures related to the extensor retinaculum

Structures superficial to the retinaculum Basilic vein

Dorsal cutaneous branch of the ulnar nerve

Cephalic vein

Superficial branch of the radial nerve

Structures passing deep to the extensor

retinaculum

Extensor carpi ulnaris tendon

Extensor digiti minimi tendon

Extensor digitorum and extensor indicis tendon

Extensor pollicis longus tendon

Extensor carpi radialis longus tendon

Abductor pollicis longus and extensor pollicis brevis

tendons

Beneath the extensor retinaculum fibrous septa form six compartments that contain

the extensor muscle tendons. Each compartment has its own synovial sheath.

The radial artery

The radial artery passes between the lateral collateral ligament of the wrist joint and

the tendons of the abductor pollicis longus and extensor pollicis brevis.

Image illustrating the topography of tendons passing under the extensor retinaculum

Page 80: MRCS preparation emrcs questions upperlimb

A 23 year old man is injured during a game of rugby. He suffers a

fracture of the distal third of his clavicle, it is a compound fracture and

there is evidence of arterial haemorrhage. Which of the following

vessels is most likely to be encountered first during subsequent

surgical exploration?

A. Posterior circumflex humeral artery

B. Axillary artery

C. Thoracoacromial artery

D. Sub scapular artery

E. Lateral thoracic artery

Next question

Similar theme in September 2011 Exam

Page 81: MRCS preparation emrcs questions upperlimb

The thoracoacromial artery arises from the second part of the axillary

artery. It is a short, wide trunk, which pierces the clavipectoral fascia,

and ends, deep to pectoralis major by dividing into four branches.

Thoracoacromial artery

The thoracoacromial artery (acromiothoracic artery; thoracic axis) is a

short trunk, which arises from the forepart of the axillary artery, its

origin being generally overlapped by the upper edge of the Pectoralis

minor.

Projecting forward to the upper border of the Pectoralis minor, it

pierces the coracoclavicular fascia and divides into four branches:

pectoral, acromial, clavicular, and deltoid.

Branch Description

Pectoral

branch

Descends between the two Pectoral muscles, and is distributed to them and to

the breast, anastomosing with the intercostal branches of the internal thoracic

artery and with the lateral thoracic.

Acromial

branch

Runs laterally over the coracoid process and under the Deltoid, to which it

gives branches; it then pierces that muscle and ends on the acromion in an

arterial network formed by branches from the suprascapular, thoracoacromial,

and posterior humeral circumflex arteries.

Clavicular

branch

Runs upwards and medially to the sternoclavicular joint, supplying this

articulation, and the Subclavius

Deltoid

branch

Arising with the acromial, it crosses over the Pectoralis minor and passes in the

same groove as the cephalic vein, between the Pectoralis major and Deltoid,

and gives branches to both muscles.

68 year old man falls onto an outstretched hand. Following the

accident he is examined in the emergency department. On palpating

his anatomical snuffbox there is tenderness noted in the base. What

is the most likely injury in this scenario?

A. Rupture of the tendon of flexor pollicis

Page 82: MRCS preparation emrcs questions upperlimb

B. Scaphoid fracture

C. Distal radius fracture

D. Rupture of flexor carpi ulnaris tendon

E. None of the above

Next question

A fall onto an outstretched hand is a common mechanism of injury for

a scaphoid fracture. This should be suspected clinically if there is

tenderness in the base of the anatomical snuffbox. A tendon rupture

would not result in bony tenderness.

Scaphoid bone

The scaphoid has a concave articular surface for the head of the

capitate and at the edge of this is a crescentic surface for the

corresponding area on the lunate.

Proximally, it has a wide convex articular surface with the radius. It

has a distally sited tubercle that can be palpated. The remaining

articular surface is to the lateral side of the tubercle. It faces laterally

and is associated with the trapezium and trapezoid bones.

The narrow strip between the radial and trapezial surfaces and the

tubercle gives rise to the radial collateral carpal ligament. The

tubercle receives part of the flexor retinaculum. This area is the only

part of the scaphoid that is available for the entry of blood vessels. It

is commonly fractured and avascular necrosis may result.

Scaphoid bone

Page 83: MRCS preparation emrcs questions upperlimb

Which of the following structures passes through the quadrangular

space near the humeral head?

A. Axillary artery

B. Radial nerve

C. Axillary nerve

D. Median nerve

E. Transverse scapular artery

Next question

The quadrangular space is bordered by the humerus laterally,

subscapularis superiorly, teres major inferiorly and the long head of

triceps medially. It lies lateral to the triangular space. It transmits the

axillary nerve and posterior circumflex humeral artery.

Page 84: MRCS preparation emrcs questions upperlimb

Image sourced from Wikipedia

Shoulder joint

Shallow synovial ball and socket type of joint.

It is an inherently unstable joint, but is capable to a wide range

of movement.

Stability is provided by muscles of the rotator cuff that pass

from the scapula to insert in the greater tuberosity (all except

sub scapularis-lesser tuberosity).

Glenoid labrum

Fibrocartilaginous rim attached to the free edge of the glenoid

cavity

Tendon of the long head of biceps arises from within the joint

from the supraglenoid tubercle, and is fused at this point to the

labrum.

The long head of triceps attaches to the infraglenoid tubercle

Fibrous capsule

Page 85: MRCS preparation emrcs questions upperlimb

Attaches to the scapula external to the glenoid labrum and to

the labrum itself (postero-superiorly)

Attaches to the humerus at the level of the anatomical neck

superiorly and the surgical neck inferiorly

Anteriorly the capsule is in contact with the tendon of

subscapularis, superiorly with the supraspinatus tendon, and

posteriorly with the tendons of infraspinatus and teres minor.

All these blend with the capsule towards their insertion.

Two defects in the fibrous capsule; superiorly for the tendon of

biceps. Anteriorly there is a defect beneath the subscapularis

tendon.

The inferior extension of the capsule is closely related to the

axillary nerve at the surgical neck and this nerve is at risk in

anteroinferior dislocations. It also means that proximally sited

osteomyelitis may progress to septic arthritis.

Movements and muscles

Flexion Anterior part of deltoid

Pectoralis major

Biceps

Coracobrachialis

Extension Posterior deltoid

Teres major

Latissimus dorsi

Adduction Pectoralis major

Latissimus dorsi

Teres major

Coracobrachialis

Abduction Mid deltoid

Supraspinatus

Medial rotation Subscapularis

Anterior deltoid

Teres major

Latissimus dorsi

Lateral rotation Posterior deltoid

Page 86: MRCS preparation emrcs questions upperlimb

Infraspinatus

Teres minor

Important anatomical relations

Anteriorly Brachial plexus

Axillary artery and vein

Posterior Suprascapular nerve

Suprascapular vessels

Inferior Axillary nerve

Circumflex humeral vessels

Which of the following structures separates the ulnar artery from the

median nerve?

A. Brachioradialis

B. Pronator teres

C. Tendon of biceps brachii

D. Flexor carpi ulnaris

E. Brachialis

Next question

It lies deep to pronator teres and this separates it from the median

nerve.

Ulnar artery

Path

Starts: middle of antecubital fossa

Passes obliquely downward, reaching the ulnar side of the

forearm at a point about midway between the elbow and the

wrist. It follows the ulnar border to the wrist, crossing over the

Page 87: MRCS preparation emrcs questions upperlimb

flexor retinaculum. It then divides into the superficial and deep

volar arches.

Relations

Deep to- Pronator teres, Flexor carpi radialis, Palmaris longus

Lies on- Brachialis and Flexor digitorum profundus

Superficial to the flexor retinaculum at the wrist

The median nerve is in relation with the medial side of the artery for

about 2.5 cm. And then crosses the vessel, being separated from it

by the ulnar head of the Pronator teres

The ulnar nerve lies medially to the lower two-thirds of the artery

Branch

Anterior interosseous artery

Page 88: MRCS preparation emrcs questions upperlimb

A 32 year old motorcyclist is involved in a road traffic accident. His

humerus is fractured and severely displaced. At the time of surgical

repair the surgeon notes that the radial nerve has been injured.

Which of the following muscles is least likely to be affected by an

injury at this site?

A. Extensor carpi radialis brevis

B. Brachioradialis

C. Abductor pollicis longus

D. Extensor pollicis brevis

E. None of the above

Next question

Page 89: MRCS preparation emrcs questions upperlimb

Muscles supplied by the radial nerve

BEST

Brachioradialis

Extensors

Supinator

Triceps

The radial nerve supplies the extensor muscles, abductor pollicis

longus and extensor pollicis brevis (the latter two being innervated by

the posterior interosseous branch of the radial nerve).

Radial nerve

Continuation of posterior cord of the brachial plexus (root values C5

to T1)

Path

In the axilla: lies posterior to the axillary artery on

subscapularis, latissimus dorsi and teres major.

Enters the arm between the brachial artery and the long head

of triceps (medial to humerus).

Spirals around the posterior surface of the humerus in the

groove for the radial nerve.

At the distal third of the lateral border of the humerus it then

pierces the intermuscular septum and descends in front of the

lateral epicondyle.

At the lateral epicondyle it lies deeply between brachialis and

brachioradialis where it then divides into a superficial and deep

terminal branch.

Deep branch crosses the supinator to become the posterior

interosseous nerve.

Page 90: MRCS preparation emrcs questions upperlimb

In the image below the relationships of the radial nerve can be

appreciated

Image sourced from Wikipedia

Regions innervated

Motor (main

nerve)

Triceps

Anconeus

Brachioradialis

Extensor carpi radialis

Motor (posterior

interosseous

branch

Extensor carpi ulnaris

Extensor digitorum

Extensor indicis

Extensor digiti minimi

Extensor pollicis longus and brevis

Abductor pollicis longus

Sensory The area of skin supplying the proximal phalanges on

the dorsal aspect of the hand is supplied by the radial

nerve (this does not apply to the little finger and part

of the ring finger)

Page 91: MRCS preparation emrcs questions upperlimb

Muscular innervation and effect of denervation

Anatomical

location

Muscle affected Effect of paralysis

Shoulder Long head of triceps Minor effects on shoulder

stability in abduction

Arm Triceps Loss of elbow extension

Forearm Supinator

Brachioradialis

Extensor carpi radialis

longus and brevis

Weakening of supination of

prone hand and elbow flexion in

mid prone position

The cutaneous sensation of the upper limb- illustrating the

contribution of the radial nerve

Image sourced from Wikipedia

Which muscle is responsible for causing flexion of the

interphalangeal joint of the thumb?

A. Flexor pollicis longus

Page 92: MRCS preparation emrcs questions upperlimb

B. Flexor pollicis brevis

C. Flexor digitorum superficialis

D. Flexor digitorum profundus

E. Adductor pollicis

Next question

There are 8 muscles:

1. Two flexors (flexor pollicis brevis and flexor pollicis longus)

2. Two extensors (extensor pollicis brevis and longus)

3. Two abductors (abductor pollicis brevis and longus)

4. One adductor (adductor pollicis)

5. One muscle that opposes the thumb by rotating the CMC joint

(opponens pollicis).

Flexor and extensor longus insert on the distal phalanx moving both

the MCP and IP joints.

Hand

Anatomy of the hand

Bones 8 Carpal bones

5 Metacarpals

14 phalanges

Intrinsic Muscles 7 Interossei - Supplied by ulnar nerve

3 palmar-adduct fingers

4 dorsal- abduct fingers

Intrinsic muscles Lumbricals

Flex MCPJ and extend the IPJ.

Origin deep flexor tendon and insertion dorsal extensor hood

mechanism.

Page 93: MRCS preparation emrcs questions upperlimb

Innervation: 1st and 2nd- median nerve, 3rd and 4th- deep

branch of the ulnar nerve.

Thenar eminence Abductor pollicis brevis

Opponens pollicis

Flexor pollicis brevis

Hypothenar

eminence

Opponens digiti minimi

Flexor digiti minimi brevis

Abductor digiti minimi

Image sourced from Wikipedia

n 18 year old man is stabbed in the axilla during a fight. His axillary

artery is lacerated and repaired. However, the surgeon neglects to

repair an associated injury to the upper trunk of the brachial plexus.

Which of the following muscles is least likely to demonstrate impaired

function as a result?

A. Palmar interossei

Page 94: MRCS preparation emrcs questions upperlimb

B. Infraspinatus

C. Brachialis

D. Supinator brevis

E. None of the above

Next question

The palmar interossei are supplied by the ulnar nerve. Which lies

inferiorly and is therefore less likely to be injured.

Brachial plexus

Origin Anterior rami of C5 to T1

Sections of the

plexus

Roots, trunks, divisions, cords, branches

Mnemonic:Real Teenagers Drink Cold Beer

Roots Located in the posterior triangle

Pass between scalenus anterior and medius

Trunks Located posterior to middle third of clavicle

Upper and middle trunks related superiorly to the subclavian

artery

Lower trunk passes over 1st rib posterior to the subclavian

artery

Divisions Apex of axilla

Cords Related to axillary artery

Diagram illustrating the branches of the brachial plexus

Page 95: MRCS preparation emrcs questions upperlimb

Image sourced from Wikipedia

Cutaneous sensation of the upper limb

Image sourced from Wikipedia

23 year old man is involved in a fight, during the dispute he sustains

a laceration to the posterior aspect of his right arm, approximately

2cm proximal to the olecranon process. On assessment in the

Page 96: MRCS preparation emrcs questions upperlimb

emergency department he is unable to extend his elbow joint. Which

of the following tendons is most likely to have been cut?

A. Triceps

B. Pronator teres

C. Brachioradialis

D. Brachialis

E. Biceps

Next question

Theme from 2009 Exam

The triceps muscle extends the elbow joint. The other muscles listed

all produce flexion of the elbow joint.

Triceps

Origin Long head- infraglenoid tubercle of the scapula.

Lateral head- dorsal surface of the humerus, lateral and proximal to

the groove of the radial nerve

Medial head- posterior surface of the humerus on the inferomedial

side of the radial groove and both of the intermuscular septae

Insertion Olecranon process of the ulna. Here the olecranon bursa is between

the triceps tendon and olecranon.

Some fibres insert to the deep fascia of the forearm, posterior capsule

of the elbow (preventing the capsule from being trapped between

olecranon and olecranon fossa during extension)

Innervation Radial nerve

Blood

supply

Profunda brachii artery

Action Elbow extension. The long head can adduct the humerus and and extend it

Page 97: MRCS preparation emrcs questions upperlimb

from a flexed position

Relations The radial nerve and profunda brachii vessels lie between the lateral and

medial heads

Which of the following muscles does not attach to the radius?

A. Pronator quadratus

B. Biceps

C. Brachioradialis

D. Supinator

E. Brachialis

Next question

The brachialis muscle inserts into the ulna. The other muscles are all

inserted onto the radius.

Radius

Bone of the forearm extending from the lateral side of the

elbow to the thumb side of the wrist

Upper end

Articular cartilage- covers medial > lateral side

Articulates with radial notch of the ulna by the annular ligament

Muscle attachment- biceps brachii at the tuberosity

Shaft

Muscle attachment-

Page 98: MRCS preparation emrcs questions upperlimb

Upper third of the body Supinator, Flexor digitorum superficialis,

Flexor pollicis longus

Middle third of the body Pronator teres

Lower quarter of the body Pronator quadratus , tendon of supinator

longus

Lower end

Quadrilateral

Anterior surface- capsule of wrist joint

Medial surface- head of ulna

Lateral surface- ends in the styloid process

Posterior surface: 3 grooves containing:

1. Tendons of extensor carpi radialis longus and brevis

2. Tendon of extensor pollicis longus

3. Tendon of extensor indicis

Page 99: MRCS preparation emrcs questions upperlimb

Which of the following is not an intrinsic muscle of the hand?

A. Opponens pollicis

B. Palmaris longus

C. Flexor pollicis brevis

D. Flexor digiti minimi brevis

E. Opponens digiti minimi

Page 100: MRCS preparation emrcs questions upperlimb

Next question

Mnemonic for intrinsic hand muscles

'A OF A OF A'

A bductor pollicis brevis

O pponens pollicis

F lexor pollicis brevis

A dductor pollicis (thenar muscles)

O pponens digiti minimi

F lexor digiti minimi brevis

A bductor digiti minimi (hypothenar muscles)

Palmaris longus originates in the forearm.

Hand

Anatomy of the hand

Bones 8 Carpal bones

5 Metacarpals

14 phalanges

Intrinsic Muscles 7 Interossei - Supplied by ulnar nerve

3 palmar-adduct fingers

4 dorsal- abduct fingers

Intrinsic muscles Lumbricals

Flex MCPJ and extend the IPJ.

Origin deep flexor tendon and insertion dorsal extensor hood

mechanism.

Innervation: 1st and 2nd- median nerve, 3rd and 4th- deep

branch of the ulnar nerve.

Thenar eminence Abductor pollicis brevis

Page 101: MRCS preparation emrcs questions upperlimb

Opponens pollicis

Flexor pollicis brevis

Hypothenar

eminence

Opponens digiti minimi

Flexor digiti minimi brevis

Abductor digiti minimi

Image sourced from Wikipedia

A 28 year old man lacerates the posterolateral aspect of his wrist with

a knife in an attempted suicide. On arrival in the emergency

department the wound is inspected and found to be located over the

lateral aspect of the extensor retinaculum (which is intact). Which of

the following structures is at greatest risk of injury?

A. Superficial branch of the radial nerve

B. Radial artery

C. Dorsal branch of the ulnar nerve

Page 102: MRCS preparation emrcs questions upperlimb

D. Tendon of extensor carpi radialis brevis

E. Tendon of extensor digiti minimi

Next question

The superficial branch of the radial nerve passes superior to the

extensor retinaculum in the position of this laceration and is at

greatest risk of injury. The dorsal branch of the ulnar nerve and artery

also pass superior to the extensor retinaculum n but are located

medially.

Extensor retinaculum

The extensor rentinaculum is a thickening of the deep fascia that

stretches across the back of the wrist and holds the long extensor

tendons in position.

Its attachments are:

The pisiform and hook of hamate medially

The end of the radius laterally

Structures related to the extensor retinaculum

Structures superficial to the

retinaculum

Basilic vein

Dorsal cutaneous branch of the ulnar nerve

Cephalic vein

Superficial branch of the radial nerve

Structures passing deep to the

extensor retinaculum

Extensor carpi ulnaris tendon

Extensor digiti minimi tendon

Extensor digitorum and extensor indicis

tendon

Extensor pollicis longus tendon

Extensor carpi radialis longus tendon

Abductor pollicis longus and extensor

pollicis brevis tendons

Page 103: MRCS preparation emrcs questions upperlimb

Beneath the extensor retinaculum fibrous septa form six

compartments that contain the extensor muscle tendons. Each

compartment has its own synovial sheath.

The radial artery

The radial artery passes between the lateral collateral ligament of the

wrist joint and the tendons of the abductor pollicis longus and

extensor pollicis brevis.

Image illustrating the topography of tendons passing under the

extensor retinaculum

ransection of the radial nerve at the level of the axilla will result in all

of the following except:

A. Loss of elbow extension.

Page 104: MRCS preparation emrcs questions upperlimb

B. Loss of extension of the interphalangeal joints.

C. Loss of metacarpophalangeal extension.

D. Loss of triceps reflex.

E. Loss of sensation overlying the first dorsal interosseous.

Next question

These may still extend by virtue of retained lumbrical muscle

function.

Radial nerve

Continuation of posterior cord of the brachial plexus (root values C5

to T1)

Path

In the axilla: lies posterior to the axillary artery on

subscapularis, latissimus dorsi and teres major.

Enters the arm between the brachial artery and the long head

of triceps (medial to humerus).

Spirals around the posterior surface of the humerus in the

groove for the radial nerve.

At the distal third of the lateral border of the humerus it then

pierces the intermuscular septum and descends in front of the

lateral epicondyle.

At the lateral epicondyle it lies deeply between brachialis and

brachioradialis where it then divides into a superficial and deep

terminal branch.

Deep branch crosses the supinator to become the posterior

interosseous nerve.

Page 105: MRCS preparation emrcs questions upperlimb

In the image below the relationships of the radial nerve can be

appreciated

Image sourced from Wikipedia

Regions innervated

Motor (main

nerve)

Triceps

Anconeus

Brachioradialis

Extensor carpi radialis

Motor (posterior

interosseous

branch

Extensor carpi ulnaris

Extensor digitorum

Extensor indicis

Extensor digiti minimi

Extensor pollicis longus and brevis

Abductor pollicis longus

Sensory The area of skin supplying the proximal phalanges on

the dorsal aspect of the hand is supplied by the radial

nerve (this does not apply to the little finger and part

of the ring finger)

Page 106: MRCS preparation emrcs questions upperlimb

Muscular innervation and effect of denervation

Anatomical

location

Muscle affected Effect of paralysis

Shoulder Long head of triceps Minor effects on shoulder

stability in abduction

Arm Triceps Loss of elbow extension

Forearm Supinator

Brachioradialis

Extensor carpi radialis

longus and brevis

Weakening of supination of

prone hand and elbow flexion in

mid prone position

The cutaneous sensation of the upper limb- illustrating the

contribution of the radial nerve

Image sourced from Wikipedia

A. Teres major

Page 107: MRCS preparation emrcs questions upperlimb

B. Pectoralis major

C. Coracobrachialis

D. Supraspinatus

E. Latissimus dorsi

Next question

Supraspinatus is an abductor of the shoulder.

Shoulder joint

Shallow synovial ball and socket type of joint.

It is an inherently unstable joint, but is capable to a wide range

of movement.

Stability is provided by muscles of the rotator cuff that pass

from the scapula to insert in the greater tuberosity (all except

sub scapularis-lesser tuberosity).

Glenoid labrum

Fibrocartilaginous rim attached to the free edge of the glenoid

cavity

Tendon of the long head of biceps arises from within the joint

from the supraglenoid tubercle, and is fused at this point to the

labrum.

The long head of triceps attaches to the infraglenoid tubercle

Fibrous capsule

Attaches to the scapula external to the glenoid labrum and to

the labrum itself (postero-superiorly)

Page 108: MRCS preparation emrcs questions upperlimb

Attaches to the humerus at the level of the anatomical neck

superiorly and the surgical neck inferiorly

Anteriorly the capsule is in contact with the tendon of

subscapularis, superiorly with the supraspinatus tendon, and

posteriorly with the tendons of infraspinatus and teres minor.

All these blend with the capsule towards their insertion.

Two defects in the fibrous capsule; superiorly for the tendon of

biceps. Anteriorly there is a defect beneath the subscapularis

tendon.

The inferior extension of the capsule is closely related to the

axillary nerve at the surgical neck and this nerve is at risk in

anteroinferior dislocations. It also means that proximally sited

osteomyelitis may progress to septic arthritis.

Movements and muscles

Flexion Anterior part of deltoid

Pectoralis major

Biceps

Coracobrachialis

Extension Posterior deltoid

Teres major

Latissimus dorsi

Adduction Pectoralis major

Latissimus dorsi

Teres major

Coracobrachialis

Abduction Mid deltoid

Supraspinatus

Medial rotation Subscapularis

Anterior deltoid

Teres major

Latissimus dorsi

Lateral rotation Posterior deltoid

Infraspinatus

Teres minor

Page 109: MRCS preparation emrcs questions upperlimb

Important anatomical relations

Anteriorly Brachial plexus

Axillary artery and vein

Posterior Suprascapular nerve

Suprascapular vessels

Inferior Axillary nerve

Circumflex humeral vessels

low?

A. External jugular

B. Axillary

C. Internal jugular

D. Azygos

E. Brachial

Next question

Cephalic vein

Path

Dorsal venous arch drains laterally into the cephalic vein

Crosses the anatomical snuffbox and travels laterally up the

arm

At the antecubital fossa connected to the basilic vein by the

median cubital vein

Pierces deep fascia of deltopectoral groove to join axillary vein

Which of the following structures is not closely related to the

brachial artery?

Page 110: MRCS preparation emrcs questions upperlimb

A. Ulnar nerve

B. Median nerve

C. Cephalic vein

D. Long head of triceps

E. Median cubital vein

Next question

The cephalic vein lies superficially and on the contralateral side

of the arm to the brachial artery. The relation of the ulnar

nerves and others are demonstrated in the image below:

Image sourced from Wikipedia

Brachial artery

The brachial artery begins at the lower border of teres major as

a continuation of the axillary artery. It terminates in the cubital

fossa at the level of the neck of the radius by dividing into the

radial and ulnar arteries.

Relations

Posterior relations include the long head of triceps with the

radial nerve and profunda vessels intervening. Anteriorly it is

overlapped by the medial border of biceps.

Page 111: MRCS preparation emrcs questions upperlimb

It is crossed by the median nerve in the middle of the arm.

In the cubital fossa it is separated from the median cubital vein

by the bicipital aponeurosis.

The basilic vein is in contact at the most proximal aspect of the

cubital fossa and lies medially.

The following statements relating to the musculocutaneous nerve are

true except?

A. It arises from the lateral cord of the brachial plexus

B. It provides cutaneous innervation to the lateral side of the

forearm

C. If damaged then extension of the elbow joint will be impaired

D. It supplies the biceps muscle

E. It runs beneath biceps

Next question

It supplies biceps, brachialis and coracobrachialis so if damaged then

elbow flexion will be impaired.

Musculocutaneous nerve

Branch of lateral cord of brachial plexus

Path

It penetrates the Coracobrachialis muscle

Passes obliquely between the Biceps brachii and the Brachialis

to the lateral side of the arm

Above the elbow it pierces the deep fascia lateral to the tendon

of the Biceps brachii

Page 112: MRCS preparation emrcs questions upperlimb

Continues into the forearm as the lateral cutaneous nerve of

the forearm

Innervates

Coracobrachialis

Biceps brachii

Brachialis

Which ligament keeps the head of the radius connected to the radial

notch of the ulna?

A. Annular (orbicular) ligament

B. Quadrate ligament

C. Radial collateral ligament of the elbow

D. Ulnar collateral ligament

E. Radial collateral ligament

Next question

The annular ligament connects the radial head to the radial notch of

the ulna. This is illustrated below:

Page 113: MRCS preparation emrcs questions upperlimb

Image sourced from Wikipedia

Radius

Bone of the forearm extending from the lateral side of the

elbow to the thumb side of the wrist

Upper end

Articular cartilage- covers medial > lateral side

Articulates with radial notch of the ulna by the annular ligament

Muscle attachment- biceps brachii at the tuberosity

Shaft

Muscle attachment-

Page 114: MRCS preparation emrcs questions upperlimb

Upper third of the body Supinator, Flexor digitorum superficialis,

Flexor pollicis longus

Middle third of the body Pronator teres

Lower quarter of the body Pronator quadratus , tendon of supinator

longus

Lower end

Quadrilateral

Anterior surface- capsule of wrist joint

Medial surface- head of ulna

Lateral surface- ends in the styloid process

Posterior surface: 3 grooves containing:

1. Tendons of extensor carpi radialis longus and brevis

2. Tendon of extensor pollicis longus

3. Tendon of extensor indicis

Page 115: MRCS preparation emrcs questions upperlimb

Image sourced from Wikipedia

A 38 year old man presents to the clinic with shoulder weakness. On

examination he has an inability to initiate shoulder abduction. Which

of the nerves listed below is least likely to be functioning normally?

A. Suprascapular nerve

B. Medial pectoral nerve

C. Axillary nerve

Page 116: MRCS preparation emrcs questions upperlimb

D. Median nerve

E. Radial nerve

Next question

Theme from April 2012 Exam

Suprascapular nerve

The suprascapular nerve arises from the upper trunk of the brachial

plexus. It lies superior to the trunks of the brachial plexus and passes

inferolaterally parallel to them. It passes through the scapular notch,

deep to trapezius. It innervates both supraspinatus and infraspinatus

and initiates abduction of the shoulder. If damaged, patients may be

able to abduct the shoulder by leaning over the affected side and

deltoid can then continue to abduct the shoulder.

Page 117: MRCS preparation emrcs questions upperlimb

Which of the following vessels provides the greatest contribution to

the arterial supply of the breast?

A. External mammary artery

B. Thoracoacromial artery

C. Internal mammary artery

D. Lateral thoracic artery

E. Subclavian artery

Page 118: MRCS preparation emrcs questions upperlimb

Next question

60% of the arterial supply to the breast is derived from the internal

mammary artery. The external mammary and lateral thoracic arteries

also make a significant (but lesser) contribution. This is of importance

clinically in performing reduction mammoplasty procedures.

Breast

The breast itself lies on a layer of pectoral fascia and the following

muscles:

1. Pectoralis major

2. Serratus anterior

3. External oblique

Image showing the topography of the female breast

Image sourced from Wikipedia

Breast anatomy

Nerve supply Branches of intercostal nerves from T4-T6.

Arterial supply Internal mammary (thoracic) artery

Page 119: MRCS preparation emrcs questions upperlimb

External mammary artery (laterally)

Anterior intercostal arteries

Thoraco-acromial artery

Venous drainage Superficial venous plexus to sub clavian, axillary and intercostal veins.

Lymphatic

drainage

70% Axillary nodes

Internal mammary chain

Other lymphatic sites such as deep cervical and supraclavicular

fossa (later in disease)

A baby is found to have a Klumpke's palsy post delivery. Which of the

following is most likely to be present?

A. Loss of flexors of the wrist

B. Weak elbow flexion

C. Pronation of the forearm

D. Adducted shoulder

E. Shoulder medially rotated

Next question

Features of Klumpkes Paralysis

Claw hand (MCP joints extended and IP joints flexed)

Loss of sensation over medial aspect of forearm and hand

Horner's syndrome

Loss of flexors of the wrist

A C8, T1 root lesion is called Klumpke's paralysis and is caused by

delivery with the arm extended.

Page 120: MRCS preparation emrcs questions upperlimb

Brachial plexus

Origin Anterior rami of C5 to T1

Sections of the

plexus

Roots, trunks, divisions, cords, branches

Mnemonic:Real Teenagers Drink Cold Beer

Roots Located in the posterior triangle

Pass between scalenus anterior and medius

Trunks Located posterior to middle third of clavicle

Upper and middle trunks related superiorly to the subclavian

artery

Lower trunk passes over 1st rib posterior to the subclavian

artery

Divisions Apex of axilla

Cords Related to axillary artery

Diagram illustrating the branches of the brachial plexus

Image sourced from Wikipedia

Cutaneous sensation of the upper limb

Page 121: MRCS preparation emrcs questions upperlimb

ith respect to the basilic vein, which statement is false?

A. Its deep anatomical location makes it unsuitable for use as an

arteriovenous access site in fistula surgery

B. It originates from the dorsal venous network on the hand

C. It travels up the medial aspect of the forearm

D. Halfway between the shoulder and the elbow it lies deep to

muscle

E. It joins the brachial vein to form the axillary vein

Next question

It is used in arteriovenous fistula surgery during a procedure known

as a basilic vein transposition.

Basilic vein

Page 122: MRCS preparation emrcs questions upperlimb

The basilic and cephalic veins both provide the main pathways of

venous drainage for the arm and hand. It is continuous with the

palmar venous arch distally and the axillary vein proximally.

Path

Originates on the medial side of the dorsal venous network of

the hand, and passes up the forearm and arm.

Most of its course is superficial.

Near the region anterior to the cubital fossa the vein joins the

cephalic vein.

Midway up the humerus the basilic vein passes deep under the

muscles.

At the lower border of the teres major muscle, the anterior and

posterior circumflex humeral veins feed into it.

Joins the brachial veins to form the axillary vein.

A 78 year old man is lifting a heavy object when a feels a pain in his

forearm and is unable to continue. He has a swelling over his upper

forearm. An MRI scan shows a small cuff of tendon still attached to

the radial tuberosity consistent with a recent tear. Which of the

following muscles has been injured?

A. Pronator teres

B. Supinator

C. Aconeus

D. Brachioradialis

E. Biceps brachii

Next question

Biceps inserts into the radial tuberosity. Distal injuries of this muscle

Page 123: MRCS preparation emrcs questions upperlimb

are rare but are reported and are clinically more important than more

proximal ruptures.

Radius

Bone of the forearm extending from the lateral side of the

elbow to the thumb side of the wrist

Upper end

Articular cartilage- covers medial > lateral side

Articulates with radial notch of the ulna by the annular ligament

Muscle attachment- biceps brachii at the tuberosity

Shaft

Muscle attachment-

Upper third of the body Supinator, Flexor digitorum superficialis,

Flexor pollicis longus

Middle third of the body Pronator teres

Lower quarter of the body Pronator quadratus , tendon of supinator

longus

Lower end

Quadrilateral

Anterior surface- capsule of wrist joint

Medial surface- head of ulna

Lateral surface- ends in the styloid process

Posterior surface: 3 grooves containing:

Page 124: MRCS preparation emrcs questions upperlimb

1. Tendons of extensor carpi radialis longus and brevis

2. Tendon of extensor pollicis longus

3. Tendon of extensor indicis

Which of the following is a branch of the third part of the axillary

artery?

A. Superior thoracic

B. Lateral thoracic

Page 125: MRCS preparation emrcs questions upperlimb

C. Dorsal scapular

D. Thoracoacromial

E. Posterior circumflex humeral

Next question

The other branches include:

Subscapular

Anterior circumflex humeral

Axilla

Boundaries of the axilla

Medially Chest wall and Serratus anterior

Laterally Humeral head

Floor Subscapularis

Anterior aspect Lateral border of Pectoralis major

Fascia Clavipectoral fascia

Content:

Long thoracic nerve (of

Bell)

Derived from C5-C7 and passes behind the brachial plexus to

enter the axilla. It lies on the medial chest wall and supplies

serratus anterior. Its location puts it at risk during axillary

surgery and damage will lead to winging of the scapula.

Thoracodorsal nerve

and thoracodorsal trunk

Innervate and vascularise latissimus dorsi.

Axillary vein Lies at the apex of the axilla, it is the continuation of the basilic

vein. Becomes the subclavian vein at the outer border of the first

rib.

Intercostobrachial Traverse the axillary lymph nodes and are often divided during

Page 126: MRCS preparation emrcs questions upperlimb

nerves axillary surgery. They provide cutaneous sensation to the axillary

skin.

Lymph nodes The axilla is the main site of lymphatic drainage for the breast.

Theme: Nerve lesions

A. Intercostobrachial

B. Median

C. Axillary

D. Radial

E. Ulnar

F. Musculocutaneous

G. Brachial plexus upper cord

H. Brachial plexus lower cord

Please select the most likely nerve injury for the scenarios given.

Each option may be used once, more than once or not at all.

268. A 23 year old rugby player sustains a Smiths Fracture. On examination

opposition of the thumb is markedly weakened.

Median

This high velocity injury can often produce significant angulation and

displacement. Both of these may impair the function of the median

nerve with loss of function of the muscles of the thenar eminence

269. A 45 year old lady recovering from a mastectomy and axillary node

clearance notices that sensation in her armpit is impaired.

Intercostobrachial

The intercostobrachial nerves are frequently injured during axillary

Page 127: MRCS preparation emrcs questions upperlimb

dissection. These nerves traverse the axilla and supply cutaneous

sensation.

270. An 8 year old boy falls onto an outstretched hand and sustains a

supracondylar fracture. In addition to a weak radial pulse the child is

noted to have loss of pronation of the affected hand.

Median

This is a common injury in children. In this case the angulation and

displacement have resulted in median nerve injury.

Next question

Brachial plexus

Origin Anterior rami of C5 to T1

Sections of the

plexus

Roots, trunks, divisions, cords, branches

Mnemonic:Real Teenagers Drink Cold Beer

Roots Located in the posterior triangle

Pass between scalenus anterior and medius

Trunks Located posterior to middle third of clavicle

Upper and middle trunks related superiorly to the subclavian

artery

Lower trunk passes over 1st rib posterior to the subclavian

artery

Divisions Apex of axilla

Cords Related to axillary artery

Diagram illustrating the branches of the brachial plexus

Page 128: MRCS preparation emrcs questions upperlimb

Image sourced from Wikipedia

Cutaneous sensation of the upper limb

Image sourced from Wikipedia

froment's test which muscle function is tested?

A. Flexor pollicis longus

Page 129: MRCS preparation emrcs questions upperlimb

B. Adductor pollicis longus

C. Abductor pollicis brevis

D. Adductor pollicis

E. Opponens pollicis

Next question

Nerve signs

Froment's sign

Assess for ulnar nerve palsy

Adductor pollicis muscle function tested

Hold a piece of paper between their thumb and index finger.

The object is then pulled away. If ulnar nerve palsy, unable to

hold the paper and will flex the flexor pollicis longus to

compensate (flexion of thumb at interphalangeal joint).

Phalen's test

Assess carpal tunnel syndrome

More sensitive than Tinel's sign

Hold wrist in maximum flexion and the test is positive if there is

numbness in the median nerve distribution.

Tinel's sign

Assess for carpal tunnel syndrome

Page 130: MRCS preparation emrcs questions upperlimb

Tap the median nerve at the wrist and the test is positive if

there is tingling/electric-like sensations over the distribution of

the median nerve.

heme: Cutaneous innervation

A. Ulnar nerve

B. Fifth cervical spinal segment

C. Radial nerve

D. Musculocutaneous nerve

E. Median nerve

F. None of these

Please select the source of innervation for the region described.

Each option may be used once, more than once or not at all.

288. The skin on the palmar aspect of the thumb

Median nerve

The median nerve supplies cutaneous sensation to this region.

See diagram below

289. The nail bed of the index finger

You answered Radial nerve

The correct answer is Median nerve

290. The skin overlying the medial aspect of the palm

Ulnar nerve

Page 131: MRCS preparation emrcs questions upperlimb

This area is innervated by the ulnar nerve.

Next question

Brachial plexus

Origin Anterior rami of C5 to T1

Sections of the

plexus

Roots, trunks, divisions, cords, branches

Mnemonic:Real Teenagers Drink Cold Beer

Roots Located in the posterior triangle

Pass between scalenus anterior and medius

Trunks Located posterior to middle third of clavicle

Upper and middle trunks related superiorly to the subclavian

artery

Lower trunk passes over 1st rib posterior to the subclavian

artery

Divisions Apex of axilla

Cords Related to axillary artery

Diagram illustrating the branches of the brachial plexus

Page 132: MRCS preparation emrcs questions upperlimb

Image sourced from Wikipedia

Cutaneous sensation of the upper limb

From which of the following foramina does the opthalmic branch of

the trigeminal nerve exit the skull?

A. Foramen ovale

Page 133: MRCS preparation emrcs questions upperlimb

B. Foramen rotundum

C. Foramen spinosum

D. Superior orbital fissure

E. Foramen magnum

Next question

Mnemonic:

Standing Room Only -Exit of branches of trigeminal nerve from the skull

V1 -Superior orbital fissure

V2 -foramen Rotundum

V3 -foramen Ovale

The opthalmic branch of the trigeminal nerve exits the skull through

the superior orbital fissure.

Trigeminal nerve

The trigeminal nerve is the main sensory nerve of the head. In

addition to its major sensory role, it also innervates the muscles of

mastication.

Distribution of the trigeminal nerve

Sensory Scalp

Face

Oral cavity (and teeth)

Nose and sinuses

Dura mater

Motor Muscles of mastication

Page 134: MRCS preparation emrcs questions upperlimb

Mylohyoid

Anterior belly of digastric

Tensor tympani

Tensor palati

Autonomic connections (ganglia) Ciliary

Sphenopalatine

Otic

Submandibular

Path

Originates at the pons

Sensory root forms the large, crescentic trigeminal ganglion

within Meckel's cave, and contains the cell bodies of incoming

sensory nerve fibres. Here the 3 branches exit.

The motor root cell bodies are in the pons and the motor fibres

are distributed via the mandibular nerve. The motor root is not

part of the trigeminal ganglion.

Branches of the trigeminal nerve

Ophthalmic nerve Sensory only

Maxillary nerve Sensory only

Mandibular nerve Sensory and motor

Sensory

Ophthalmic Exits skull via the superior orbital fissure

Sensation of: scalp and forehead, the upper eyelid, the conjunctiva and

cornea of the eye, the nose (including the tip of the nose, except alae nasi),

the nasal mucosa, the frontal sinuses, and parts of the meninges (the dura and

blood vessels).

Maxillary

nerve

Exit skull via the foramen rotundum

Sensation: lower eyelid and cheek, the nares and upper lip, the upper teeth

Page 135: MRCS preparation emrcs questions upperlimb

and gums, the nasal mucosa, the palate and roof of the pharynx, the

maxillary, ethmoid and sphenoid sinuses, and parts of the meninges.

Mandibular

nerve

Exit skull via the foramen ovale

Sensation: lower lip, the lower teeth and gums, the chin and jaw (except the

angle of the jaw), parts of the external ear, and parts of the meninges.

Motor

Distributed via the mandibular nerve.

The following muscles of mastication are innervated:

Masseter

Temporalis

Medial pterygoid

Lateral pterygoid

Other muscles innervated include:

Tensor veli palatini

Mylohyoid

Anterior belly of digastric

Tensor tympani

Rate

question:

1

2

3

4

5

Next question

Comment on this question

42 year old lady has had an axillary node clearance for breast malignancy. Post

operatively she reports weakness of the shoulder. She is unable to push herself

forwards from a wall with the right arm and the scapula is pushed out medially from

the chest wall. What is the most likely nerve injury?

Page 136: MRCS preparation emrcs questions upperlimb

A. C5, C6

B. C8, T1

C. Axillary nerve

D. Long thoracic nerve

E. Spinal accessory nerve

Theme from January 2012 and 2009 Exam

The patient has a winged scapula caused by damage to the long thoracic nerve

(C5,6,7) during surgery. The long thoracic nerve innervates serratus anterior. Serratus

anterior causes pushing out of the scapula during a punch.

NB winging of the scapular laterally may indicate trapezius muscle weakness.

Innervated by the spinal accessory nerve.

Long thoracic nerve

Derived from ventral rami of C5, C6, and C7 (close to their emergence from

intervertebral foramina)

It runs downward and passes either anterior or posterior to the middle scalene

muscle

It reaches upper tip of serratus anterior muscle and descends on outer surface

of this muscle, giving branches into it

Winging of Scapula occurs in long thoracic nerve injury (most common) or

from spinal accessory nerve injury (which denervates the trapezius) or a dorsal

scapular nerve injury

A 36 year old male is admitted for elective surgery for a lymph node biopsy in the

supraclavicular region. Post operatively the patient has difficulty shrugging his left shoulder.

What nerve has been damaged?

A. Phrenic nerve

B. Axillary nerve

C. C5, C6 lesion

D. C8, T1 lesion

Page 137: MRCS preparation emrcs questions upperlimb

E. Accessory nerve

Theme from September 2011 Exam

The accessory nerve lies in the posterior triangle and may be injured in this region. Apart

from problems with shrugging the shoulder, he may also have difficulty lifting his arm above

his head.

Posterior triangle of the neck

Boundaries

Apex Sternocleidomastoid and the Trapezius muscles at the Occipital bone

Anterior Posterior border of the Sternocleidomastoid

Posterior Anterior border of the Trapezius

Base Middle third of the clavicle

Image sourced from Wikipedia

Contents

Nerves Accessory nerve Phrenic nerve Three trunks of the brachial plexus Branches of the cervical plexus: Supraclavicular nerve, transverse cervical

Page 138: MRCS preparation emrcs questions upperlimb

nerve, great auricular nerve, lesser occipital nerve

Vessels External jugular vein Subclavian artery

Muscles Inferior belly of omohyoid Scalene

Lymph

nodes

Supraclavicular Occipital

Which of the following muscles is supplied by the musculocutaneous nerve?

A. Brachialis

B. Latissimus dorsi

C. Flexor carpi ulnaris

D. Teres minor

E. Triceps

Mnemonic

Muscles innervated by the

musculocutaneous nerve BBC:

Biceps brachii

Brachialis

Coracobrachialis

Musculocutaneous nerve

Branch of lateral cord of brachial plexus

Path

It penetrates the Coracobrachialis muscle

Passes obliquely between the Biceps brachii and the Brachialis to the lateral

side of the arm

Above the elbow it pierces the deep fascia lateral to the tendon of the Biceps

brachii

Continues into the forearm as the lateral cutaneous nerve of the forearm

Page 139: MRCS preparation emrcs questions upperlimb

Innervates

Coracobrachialis

Biceps brachii

Brachialis

A 17 year old male presents to the clinic. He complains of difficulty using his left hand. It has

been a persistent problem since he sustained a distal humerus fracture as a child. On

examination there is diminished sensation overlying the hypothenar eminence and medial

one and half fingers. What is the most likely nerve lesion?

A. Anterior interosseous nerve

B. Posterior interosseous nerve

C. Ulnar nerve

D. Median nerve

E. Radial nerve

Theme from April 2012 Exam

This sensory deficit pattern is most consistent with ulnar nerve injury.

Page 140: MRCS preparation emrcs questions upperlimb

Image sourced from Wikipedia

Ulnar nerve

Origin

C8, T1

Supplies (no muscles in the upper arm)

Flexor carpi ulnaris Flexor digitorum profundus Flexor digiti minimi Abductor digiti minimi Opponens digiti minimi Adductor pollicis Interossei muscle Third and fourth lumbricals

Page 141: MRCS preparation emrcs questions upperlimb

Palmaris brevis

Path

Posteromedial aspect of ulna to flexor compartment of forearm, then along the ulnar. Passes beneath the flexor carpi ulnaris muscle, then superficially through the flexor retinaculum into the palm of the hand.

Image sourced from Wikipedia

Branches

Branch Supplies

Articular branch Flexor carpi ulnaris

Medial half of the flexor digitorum profundus

Palmar cutaneous branch (Arises near the Skin on the medial part of the palm

Page 142: MRCS preparation emrcs questions upperlimb

middle of the forearm)

Dorsal cutaneous branch Dorsal surface of the medial part of the hand

Superficial branch Cutaneous fibres to the anterior surfaces of the

medial one and one-half digits

Deep branch Hypothenar muscles

All the interosseous muscles

Third and fourth lumbricals

Adductor pollicis

Medial head of the flexor pollicis brevis

Effects of injury

Damage at the wrist Wasting and paralysis of intrinsic hand muscles (claw hand) Wasting and paralysis of hypothenar muscles Loss of sensation medial 1 and half fingers

Damage at the

elbow

Radial deviation of the wrist Clawing less in 3rd and 4th digits

A 72 year old male with end stage critical ischaemia is undergoing an axillo-femoral

bypass. What structure is not closely related to the axillary artery?

A. Posterior cord of the brachial plexus

B. Scalenus anterior muscle

C. Pectoralis minor muscle

D. Axillary vein

E. Lateral cord of the brachial plexus

The axillary artery is the continuation of the subclavian artery. It is surrounded by the

cords of the brachial plexus (from which they are named). The axillary vein runs

alongside the axillary artery throughout its length.

Axilla

Boundaries of the axilla

Medially Chest wall and Serratus anterior

Laterally Humeral head

Page 143: MRCS preparation emrcs questions upperlimb

Floor Subscapularis

Anterior aspect Lateral border of Pectoralis major

Fascia Clavipectoral fascia

Content:

Long thoracic nerve (of

Bell)

Derived from C5-C7 and passes behind the brachial plexus

to enter the axilla. It lies on the medial chest wall and

supplies serratus anterior. Its location puts it at risk during

axillary surgery and damage will lead to winging of the

scapula.

Thoracodorsal nerve

and thoracodorsal

trunk

Innervate and vascularise latissimus dorsi.

Axillary vein Lies at the apex of the axilla, it is the continuation of the

basilic vein. Becomes the subclavian vein at the outer

border of the first rib.

Intercostobrachial

nerves

Traverse the axillary lymph nodes and are often divided

during axillary surgery. They provide cutaneous sensation

to the axillary skin.

Lymph nodes The axilla is the main site of lymphatic drainage for the

breast.

Which of the following carpal bones is a sesamoid bone in the tendon of flexor carpi ulnaris?

A. Triquetrum

B. Lunate

C. Pisiform

D. Scaphoid

E. Capitate

This small bone has a single articular facet. It projects from the triquetral bone at the ulnar

aspect of the wrist where most regard it as a sesamoid bine lying within the tendon of flexor

carpi ulnaris.

Carpal bones

Diagrammatic image of carpal bones

Page 144: MRCS preparation emrcs questions upperlimb

Image sourced from Wikipedia

Key to image

A Scaphoid

B Lunate

C Triquetrum

D Pisiform

E Trapezium

F Trapezoid

G Capitate

H Hamate

1 Radius

2 Ulna

3 Metacarpals

No tendons attach to: Scaphoid, lunate, triquetrum (stabilised by ligaments)

Page 145: MRCS preparation emrcs questions upperlimb

A 70 year old man falls and fractures his scaphoid bone. The fracture is displaced

and the decision is made to insert a screw to fix the fracture. Which of the following

structures lies directly medial to the scaphoid?

A. Lunate

B. Pisiform

C. Trapezoid

D. Trapezium

E. None of the above

The lunate lies medially in the anatomical plane. Fractures of the scaphoid that are

associated with high velocity injuries may cause associated lunate dislocation.

Scaphoid bone

The scaphoid has a concave articular surface for the head of the capitate and at the

edge of this is a crescentic surface for the corresponding area on the lunate.

Proximally, it has a wide convex articular surface with the radius. It has a distally

sited tubercle that can be palpated. The remaining articular surface is to the lateral

side of the tubercle. It faces laterally and is associated with the trapezium and

trapezoid bones.

The narrow strip between the radial and trapezial surfaces and the tubercle gives

rise to the radial collateral carpal ligament. The tubercle receives part of the flexor

retinaculum. This area is the only part of the scaphoid that is available for the entry

of blood vessels. It is commonly fractured and avascular necrosis may result.

Scaphoid bone

Page 146: MRCS preparation emrcs questions upperlimb

Image sourced from Wikipedia

A 73 year old lady is hit by a car. She suffers a complex fracture of the distal aspect of her

humerus with associated injury to the radial nerve. Which of the following movements will

be most impaired as a result?

A. Elbow extension

B. Elbow flexion

C. Shoulder abduction

D. Wrist extension

E. None of the above

The triceps will not be affected so elbow extension will be preserved. Loss of wrist extension

will be the most obvious effect.

Radial nerve

Continuation of posterior cord of the brachial plexus (root values C5 to T1)

Path

In the axilla: lies posterior to the axillary artery on subscapularis, latissimus dorsi and teres major.

Page 147: MRCS preparation emrcs questions upperlimb

Enters the arm between the brachial artery and the long head of triceps (medial to humerus).

Spirals around the posterior surface of the humerus in the groove for the radial nerve.

At the distal third of the lateral border of the humerus it then pierces the intermuscular septum and descends in front of the lateral epicondyle.

At the lateral epicondyle it lies deeply between brachialis and brachioradialis where it then divides into a superficial and deep terminal branch.

Deep branch crosses the supinator to become the posterior interosseous nerve.

In the image below the relationships of the radial nerve can be appreciated

Image sourced from Wikipedia

Regions innervated

Motor (main nerve) Triceps Anconeus Brachioradialis Extensor carpi radialis

Motor (posterior

interosseous

branch

Extensor carpi ulnaris Extensor digitorum Extensor indicis Extensor digiti minimi Extensor pollicis longus and brevis Abductor pollicis longus

Page 148: MRCS preparation emrcs questions upperlimb

Sensory The area of skin supplying the proximal phalanges on the dorsal aspect

of the hand is supplied by the radial nerve (this does not apply to the

little finger and part of the ring finger)

Muscular innervation and effect of denervation

Anatomical

location

Muscle affected Effect of paralysis

Shoulder Long head of triceps Minor effects on shoulder stability in

abduction

Arm Triceps Loss of elbow extension

Forearm Supinator

Brachioradialis

Extensor carpi radialis

longus and brevis

Weakening of supination of prone hand and

elbow flexion in mid prone position

The cutaneous sensation of the upper limb- illustrating the contribution of the radial nerve

Image sourced from Wikipedia

Page 149: MRCS preparation emrcs questions upperlimb

An 18 year old man develops a severe spreading sepsis of the hand. The palm is explored

surgically and the flexor digiti minimi brevis muscle is mobilised to facilitate drainage of the

infection. Which of the following structures is not closely related to this muscle?

A. The hook of hamate

B. Median nerve

C. Superficial palmar arterial arch

D. Digital nerves arising from the ulnar nerve

E. None of the above

The flexor digiti minimi brevis originates from the Hamate, on its undersurface lie the ulnar

contribution to the superficial palmar arterial arch and digital nerves derived from the ulnar

nerve. The median nerve overlies the flexor tendons.

Hand

Anatomy of the hand

Bones 8 Carpal bones 5 Metacarpals 14 phalanges

Intrinsic Muscles 7 Interossei - Supplied by ulnar nerve

3 palmar-adduct fingers 4 dorsal- abduct fingers

Intrinsic muscles Lumbricals

Flex MCPJ and extend the IPJ. Origin deep flexor tendon and insertion dorsal extensor hood

mechanism. Innervation: 1st and 2nd- median nerve, 3rd and 4th- deep

branch of the ulnar nerve.

Thenar eminence Abductor pollicis brevis Opponens pollicis Flexor pollicis brevis

Page 150: MRCS preparation emrcs questions upperlimb

Hypothenar

eminence

Opponens digiti minimi Flexor digiti minimi brevis Abductor digiti minimi

Image sourced from Wikipedia

A 22 year old man develops an infection in the pulp of his little finger. What is the most

proximal site to which this infection may migrate?

A. The metacarpophalangeal joint

B. The distal interphalangeal joint

C. The proximal interphalangeal joint

D. Proximal to the flexor retinaculum

E. Immediately distal to the carpal tunnel

Page 151: MRCS preparation emrcs questions upperlimb

The 5th tendon sheath extends from the little finger to the proximal aspect of the carpal

tunnel. This carries a significant risk of allowing infections to migrate proximally.

Hand

Anatomy of the hand

Bones 8 Carpal bones 5 Metacarpals 14 phalanges

Intrinsic Muscles 7 Interossei - Supplied by ulnar nerve

3 palmar-adduct fingers 4 dorsal- abduct fingers

Intrinsic muscles Lumbricals

Flex MCPJ and extend the IPJ. Origin deep flexor tendon and insertion dorsal extensor hood

mechanism. Innervation: 1st and 2nd- median nerve, 3rd and 4th- deep

branch of the ulnar nerve.

Thenar eminence Abductor pollicis brevis Opponens pollicis Flexor pollicis brevis

Hypothenar

eminence

Opponens digiti minimi Flexor digiti minimi brevis Abductor digiti minimi

Page 152: MRCS preparation emrcs questions upperlimb

Image sourced from Wikipedia

Which of the following muscles is not innervated by the deep branch of the ulnar

nerve?

A. Adductor pollicis

B. Hypothenar muscles

C. All the interosseous muscles

D. Opponens pollicis

E. Third and fourth lumbricals

Ulnar nerve

Origin

C8, T1

Supplies (no muscles in the upper arm)

Page 153: MRCS preparation emrcs questions upperlimb

Flexor carpi ulnaris

Flexor digitorum profundus

Flexor digiti minimi

Abductor digiti minimi

Opponens digiti minimi

Adductor pollicis

Interossei muscle

Third and fourth lumbricals

Palmaris brevis

Path

Posteromedial aspect of ulna to flexor compartment of forearm, then along the

ulnar. Passes beneath the flexor carpi ulnaris muscle, then superficially

through the flexor retinaculum into the palm of the hand.

Branches

Branch Supplies

Articular branch Flexor carpi ulnaris

Medial half of the flexor digitorum profundus

Palmar cutaneous branch (Arises near Skin on the medial part of the palm

Page 154: MRCS preparation emrcs questions upperlimb

the middle of the forearm)

Dorsal cutaneous branch Dorsal surface of the medial part of the hand

Superficial branch Cutaneous fibres to the anterior surfaces of

the medial one and one-half digits

Deep branch Hypothenar muscles

All the interosseous muscles

Third and fourth lumbricals

Adductor pollicis

Medial head of the flexor pollicis brevis

Effects of injury

Damage at the

wrist

Wasting and paralysis of intrinsic hand muscles (claw

hand)

Wasting and paralysis of hypothenar muscles

Loss of sensation medial 1 and half fingers

Damage at the

elbow

Radial deviation of the wrist

Clawing less in 3rd and 4th digits

Which of the following structures lie between the lateral and medial heads of the

triceps muscle?

A. Radial nerve

B. Median nerve

C. Ulnar nerve

D. Axillary nerve

E. Medial cutaneous nerve of the forearm

The radial nerve runs in its groove on between the two heads. The ulnar nerve lies

anterior to the medial head. The axillary nerve passes through the quadrangular space.

This lies superior to lateral head of the triceps muscle and thus the lateral border of

the quadrangular space is the humerus. Therefore the correct answer is the radial

nerve.

Triceps

Origin Long head- infraglenoid tubercle of the scapula.

Lateral head- dorsal surface of the humerus, lateral and proximal

to the groove of the radial nerve

Medial head- posterior surface of the humerus on the

inferomedial side of the radial groove and both of the

Page 155: MRCS preparation emrcs questions upperlimb

intermuscular septae

Insertion Olecranon process of the ulna. Here the olecranon bursa is

between the triceps tendon and olecranon.

Some fibres insert to the deep fascia of the forearm, posterior

capsule of the elbow (preventing the capsule from being trapped

between olecranon and olecranon fossa during extension)

Innervation Radial nerve

Blood

supply

Profunda brachii artery

Action Elbow extension. The long head can adduct the humerus and and extend

it from a flexed position

Relations The radial nerve and profunda brachii vessels lie between the lateral and

medial heads

Into which of the following structures does the superior part of the fibrous capsule of

the shoulder joint insert?

A. The surgical neck of the humerus

B. The body of the humerus

C. The bicipital groove

D. Immediately distal to the greater tuberosity

E. The anatomical neck of the humerus

The shoulder joint is a shallow joint, hence its great mobility. However, this comes at

the expense of stability. The fibrous capsule attaches to the anatomical neck

superiorly and the surgical neck inferiorly

Shoulder joint

Shallow synovial ball and socket type of joint.

It is an inherently unstable joint, but is capable to a wide range of movement.

Stability is provided by muscles of the rotator cuff that pass from the scapula

to insert in the greater tuberosity (all except sub scapularis-lesser tuberosity).

Glenoid labrum

Fibrocartilaginous rim attached to the free edge of the glenoid cavity

Tendon of the long head of biceps arises from within the joint from the

supraglenoid tubercle, and is fused at this point to the labrum.

The long head of triceps attaches to the infraglenoid tubercle

Page 156: MRCS preparation emrcs questions upperlimb

Fibrous capsule

Attaches to the scapula external to the glenoid labrum and to the labrum itself

(postero-superiorly)

Attaches to the humerus at the level of the anatomical neck superiorly and the

surgical neck inferiorly

Anteriorly the capsule is in contact with the tendon of subscapularis,

superiorly with the supraspinatus tendon, and posteriorly with the tendons of

infraspinatus and teres minor. All these blend with the capsule towards their

insertion.

Two defects in the fibrous capsule; superiorly for the tendon of biceps.

Anteriorly there is a defect beneath the subscapularis tendon.

The inferior extension of the capsule is closely related to the axillary nerve at

the surgical neck and this nerve is at risk in anteroinferior dislocations. It also

means that proximally sited osteomyelitis may progress to septic arthritis.

Movements and muscles

Flexion Anterior part of deltoid

Pectoralis major

Biceps

Coracobrachialis

Extension Posterior deltoid

Teres major

Latissimus dorsi

Adduction Pectoralis major

Latissimus dorsi

Teres major

Coracobrachialis

Abduction Mid deltoid

Supraspinatus

Medial rotation Subscapularis

Anterior deltoid

Teres major

Latissimus dorsi

Lateral rotation Posterior deltoid

Infraspinatus

Teres minor

Important anatomical relations

Anteriorly Brachial plexus

Axillary artery and vein

Posterior Suprascapular nerve

Suprascapular vessels

Inferior Axillary nerve

Circumflex humeral vessels

سؤال غلط

Page 157: MRCS preparation emrcs questions upperlimb

Damage to the posterior cord of the brachial plexus will not result in any of the following

except:

A. Klumpkes palsy

B. Anaesthesia overlying the lateral aspect of the forearm

C. A warm sweaty hand on the affected side

D. Loss of flexion of the arm

E. Anaesthesia overlying the posterior surface of the arm

The radial nerve gives cutaneous branches which supply the forearm posteriorly and the arm

laterally. Division of the posterior cord will impair the upper level of cutaneous sensation.

However, the lateral cutaneous nerve of the forearm arises from the musculocutaneous

nerve and would be unaffected. Loss of sympathetic function would not result in a sweaty

hand. Klumpkes palsy occurs when the lower roots are C8-T1 are damaged.

Brachial plexus

Origin Anterior rami of C5 to T1

Sections of the

plexus

Roots, trunks, divisions, cords, branches Mnemonic:Real Teenagers Drink Cold Beer

Roots Located in the posterior triangle Pass between scalenus anterior and medius

Trunks Located posterior to middle third of clavicle Upper and middle trunks related superiorly to the subclavian

artery Lower trunk passes over 1st rib posterior to the subclavian

artery

Divisions Apex of axilla

Cords Related to axillary artery

Diagram illustrating the branches of the brachial plexus

Page 158: MRCS preparation emrcs questions upperlimb

Image sourced from Wikipedia

Cutaneous sensation of the upper limb

Image sourced from Wikipedia

A woman develops winging of the scapula following a Patey mastectomy. What is the

most likely cause?

Page 159: MRCS preparation emrcs questions upperlimb

A. Division of pectoralis minor to access level 3 axillary nodes

B. Damage to the brachial plexus during axillary dissection

C. Damage to the long thoracic nerve during axillary dissection

D. Division of the thoracodorsal trunk during axillary dissection

E. Damage to the thoracodorsal trunk during axillary dissection

Theme from January 2012 exam

The serratus anterior muscle is supplied by the long thoracic nerve which runs along

the surface of serratus anterior and is liable to injury during nodal dissection.

Although pectoralis minor is divided during a Patey mastectomy (now seldom

performed) it is rare for this alone to produce winging of the scapula.

Long thoracic nerve

Derived from ventral rami of C5, C6, and C7 (close to their emergence from

intervertebral foramina)

It runs downward and passes either anterior or posterior to the middle scalene

muscle

It reaches upper tip of serratus anterior muscle and descends on outer surface

of this muscle, giving branches into it

Winging of Scapula occurs in long thoracic nerve injury (most common) or

from spinal accessory nerve injury (which denervates the trapezius) or a dorsal

scapular nerve injury

Which of the following is not closely related to the capitate bone?

A. Lunate bone

B. Scaphoid bone

C. Ulnar nerve

D. Hamate bone

E. Trapezoid bone

The ulnar nerve and artery lie adjacent to the pisiform bone. The capitate bone

articulates with the lunate, scaphoid, hamate and trapezoid bones, which are

therefore closely related to it.

Capitate bone

Page 160: MRCS preparation emrcs questions upperlimb

This is the largest of the carpal bones. It is centrally placed with a rounded

head set into the cavities of the lunate and scaphoid bones. Flatter articular

surfaces are present for the hamate medially and the trapezoid laterally.

Distally the bone articulates predominantly with the middle metacarpal.

An injury to the spinal accessory nerve will affect which of the following

movements?

A. Lateral rotation of the arm

B. Adduction of the arm at the glenohumeral joint

C. Protraction of the scapula

D. Upward rotation of the scapula

E. Depression of the scapula

The spinal accessory nerve innervates trapezius. The entire muscle will retract the

scapula. However, its upper and lower fibres act together to upwardly rotate it.

Shoulder joint

Shallow synovial ball and socket type of joint.

It is an inherently unstable joint, but is capable to a wide range of movement.

Stability is provided by muscles of the rotator cuff that pass from the scapula

to insert in the greater tuberosity (all except sub scapularis-lesser tuberosity).

Glenoid labrum

Fibrocartilaginous rim attached to the free edge of the glenoid cavity

Tendon of the long head of biceps arises from within the joint from the

supraglenoid tubercle, and is fused at this point to the labrum.

The long head of triceps attaches to the infraglenoid tubercle

Fibrous capsule

Attaches to the scapula external to the glenoid labrum and to the labrum itself

(postero-superiorly)

Attaches to the humerus at the level of the anatomical neck superiorly and the

surgical neck inferiorly

Anteriorly the capsule is in contact with the tendon of subscapularis,

superiorly with the supraspinatus tendon, and posteriorly with the tendons of

infraspinatus and teres minor. All these blend with the capsule towards their

insertion.

Page 161: MRCS preparation emrcs questions upperlimb

Two defects in the fibrous capsule; superiorly for the tendon of biceps.

Anteriorly there is a defect beneath the subscapularis tendon.

The inferior extension of the capsule is closely related to the axillary nerve at

the surgical neck and this nerve is at risk in anteroinferior dislocations. It also

means that proximally sited osteomyelitis may progress to septic arthritis.

Movements and muscles

Flexion Anterior part of deltoid

Pectoralis major

Biceps

Coracobrachialis

Extension Posterior deltoid

Teres major

Latissimus dorsi

Adduction Pectoralis major

Latissimus dorsi

Teres major

Coracobrachialis

Abduction Mid deltoid

Supraspinatus

Medial rotation Subscapularis

Anterior deltoid

Teres major

Latissimus dorsi

Lateral rotation Posterior deltoid

Infraspinatus

Teres minor

Important anatomical relations

Anteriorly Brachial plexus

Axillary artery and vein

Posterior Suprascapular nerve

Suprascapular vessels

Inferior Axillary nerve

Circumflex humeral vessels

A 23 year old man falls over whilst intoxicated and a shard of glass transects his median

nerve at the proximal border of the flexor retinaculum. His tendons escape injury. Which of

the following features will not be present?

A. Weakness of thumb abduction

B. Loss of sensation on the dorsal aspect of the thenar eminence

Page 162: MRCS preparation emrcs questions upperlimb

C. Loss of power of opponens pollicis

D. Adduction and lateral rotation of the thumb at rest

E. Loss of power of abductor pollicis brevis

The median nerve may be injured proximal to the flexor retinaculum. This will result in loss

of flexor pollicis brevis, opponens pollicis and the first and second lumbricals. When the

patient is asked to close the hand slowly there is a lag of the index and middle fingers

reflecting the impaired lumbrical muscle function. The sensory changes are minor and do

not extend to the dorsal aspect of the thenar eminence.

Abductor pollicis longus will contribute to thumb abduction (and is innervated by the

posterior interosseous nerve) and therefore abduction will be weaker than prior to the

injury.

Median nerve

The median nerve is formed by the union of a lateral and medial root respectively from the

lateral (C5,6,7) and medial (C8 and T1) cords of the brachial plexus; the medial root passes

anterior to the third part of the axillary artery. The nerve descends lateral to the brachial

artery, crosses to its medial side (usually passing anterior to the artery). It passes deep to the

bicipital aponeurosis and the median cubital vein at the elbow.

It passes between the two heads of the pronator teres muscle, and runs on the deep surface

of flexor digitorum superficialis (within its fascial sheath).

Near the wrist it becomes superficial between the tendons of flexor digitorum superficialis

and flexor carpi radialis, deep to palmaris longus tendon. It passes deep to the flexor

retinaculum to enter the palm, but lies anterior to the long flexor tendons within the carpal

tunnel.

Branches

Region Branch

Upper arm No branches, although the nerve commonly communicates with the

musculocutaneous nerve

Forearm Pronator teres

Flexor carpi radialis

Palmaris longus

Flexor digitorum superficialis

Flexor pollicis longus

Flexor digitorum profundus (only the radial half)

Page 163: MRCS preparation emrcs questions upperlimb

Distal

forearm

Palmar cutaneous branch

Hand

(Motor)

Motor supply (LOAF)

Lateral 2 lumbricals Opponens pollicis Abductor pollicis brevis Flexor pollicis brevis

Hand

(Sensory)

Over thumb and lateral 2 ½ fingers On the palmar aspect this projects proximally, on the dorsal aspect only

the distal regions are innervated with the radial nerve providing the more proximal cutaneous innervation.

Patterns of damage

Damage at wrist

e.g. carpal tunnel syndrome paralysis and wasting of thenar eminence muscles and opponens pollicis (ape hand

deformity) sensory loss to palmar aspect of lateral (radial) 2 ½ fingers

Damage at elbow, as above plus:

unable to pronate forearm weak wrist flexion ulnar deviation of wrist

Anterior interosseous nerve (branch of median nerve)

leaves just below the elbow results in loss of pronation of forearm and weakness of long flexors of thumb and

index finger

Topography of the median nerve

Page 164: MRCS preparation emrcs questions upperlimb

Image sourced from Wikipedia

A 23 year old man falls and injures his hand. There are concerns that he may have a scaphoid

fracture as there is tenderness in his anatomical snuffbox on clinical examination. Which of

the following forms the posterior border of this structure?

A. Basilic vein

B. Radial artery

C. Extensor pollicis brevis

D. Abductor pollicis longus

E. Extensor pollicis longus

Theme from 2009 Exam

Page 165: MRCS preparation emrcs questions upperlimb

Theme from September 2012 Exam

Its boundaries are extensor pollicis longus, medially (posterior border) and laterally (anterior

border) by the tendons of abductor pollicis longus and extensor pollicis brevis.

Anatomical snuffbox

Posterior border Tendon of extensor pollicis longus

Anterior border Tendons of extensor pollicis brevis and abductor pollicis longus

Proximal border Styloid process of the radius

Distal border Apex of snuffbox triangle

Floor Trapezium and scaphoid

Content Radial artery

Image showing the anatomical snuffbox

Image sourced from Wikipedia

A 28 year old man is stabbed outside a nightclub in the upper arm. The median nerve is

transected. Which of the following muscles will demonstrate impaired function as a result?

A. Palmaris brevis

B. Second and third interossei

Page 166: MRCS preparation emrcs questions upperlimb

C. Adductor pollicis

D. Abductor pollicis longus

E. Abductor pollicis brevis

Palmaris brevis - Ulnar nerve

Palmar interossei- Ulnar nerve

Adductor pollicis - Ulnar nerve

Abductor pollicis longus - Posterior

interosseous nerve

Abductor pollicis brevis - Median

nerve

The median nerve innervates all the short muscles of the thumb except the adductor and

the deep head of the short flexor. Palmaris and the interossei are innervated by the ulnar

nerve.

Median nerve

The median nerve is formed by the union of a lateral and medial root respectively from the

lateral (C5,6,7) and medial (C8 and T1) cords of the brachial plexus; the medial root passes

anterior to the third part of the axillary artery. The nerve descends lateral to the brachial

artery, crosses to its medial side (usually passing anterior to the artery). It passes deep to the

bicipital aponeurosis and the median cubital vein at the elbow.

It passes between the two heads of the pronator teres muscle, and runs on the deep surface

of flexor digitorum superficialis (within its fascial sheath).

Near the wrist it becomes superficial between the tendons of flexor digitorum superficialis

and flexor carpi radialis, deep to palmaris longus tendon. It passes deep to the flexor

retinaculum to enter the palm, but lies anterior to the long flexor tendons within the carpal

tunnel.

Branches

Region Branch

Upper arm No branches, although the nerve commonly communicates with the

musculocutaneous nerve

Forearm Pronator teres

Flexor carpi radialis

Palmaris longus

Flexor digitorum superficialis

Page 167: MRCS preparation emrcs questions upperlimb

Flexor pollicis longus

Flexor digitorum profundus (only the radial half)

Distal

forearm

Palmar cutaneous branch

Hand

(Motor)

Motor supply (LOAF)

Lateral 2 lumbricals Opponens pollicis Abductor pollicis brevis Flexor pollicis brevis

Hand

(Sensory)

Over thumb and lateral 2 ½ fingers On the palmar aspect this projects proximally, on the dorsal aspect only

the distal regions are innervated with the radial nerve providing the more proximal cutaneous innervation.

Patterns of damage

Damage at wrist

e.g. carpal tunnel syndrome paralysis and wasting of thenar eminence muscles and opponens pollicis (ape hand

deformity) sensory loss to palmar aspect of lateral (radial) 2 ½ fingers

Damage at elbow, as above plus:

unable to pronate forearm weak wrist flexion ulnar deviation of wrist

Anterior interosseous nerve (branch of median nerve)

leaves just below the elbow results in loss of pronation of forearm and weakness of long flexors of thumb and

index finger

Topography of the median nerve

Page 168: MRCS preparation emrcs questions upperlimb

Image sourced from Wikipedia

Which of the following is not a branch of the posterior cord of the brachial plexus?

A. Thoracodorsal nerve

B. Axillary nerve

C. Radial nerve

D. Lower subscapular nerve

E. Musculocutaneous nerve

Mnemonic branches off the

posterior cord

Page 169: MRCS preparation emrcs questions upperlimb

S ubscapular (upper and

lower)

T horacodorsal

A xillary

R adial

The musculocutaneous nerve is a branch off the lateral cord.

Brachial plexus

Origin Anterior rami of C5 to T1

Sections of the

plexus

Roots, trunks, divisions, cords, branches Mnemonic:Real Teenagers Drink Cold Beer

Roots Located in the posterior triangle Pass between scalenus anterior and medius

Trunks Located posterior to middle third of clavicle Upper and middle trunks related superiorly to the subclavian

artery Lower trunk passes over 1st rib posterior to the subclavian

artery

Divisions Apex of axilla

Cords Related to axillary artery

Diagram illustrating the branches of the brachial plexus

Page 170: MRCS preparation emrcs questions upperlimb

Image sourced from Wikipedia

Cutaneous sensation of the upper limb

Image sourced from Wikipedia

A 56 year old machinist has his arm entrapped in a steel grinder and is brought to the

emergency department. On examination, he is unable to extend his

Page 171: MRCS preparation emrcs questions upperlimb

metacarpophalangeal joints and abduct his shoulder. He has weakness of his elbow

and wrist. What has been injured?

A. Ulnar nerve

B. Axillary nerve

C. Medial cord of brachial plexus

D. Lateral cord of brachial plexus

E. Posterior cord of brachial plexus

The posterior cord gives rise to:

Radial nerve ((innervates the triceps, brachioradialis, wrist extensors, and

finger extensors)

Axillary nerve (innervates deltoid and teres minor)

Upper subscapular nerve (innervates subscapularis)

Lower subscapular nerve (innervates teres major and subscapularis)

Thoracodorsal nerve (innervates latissimus dorsi)

Theme from September 2012 exam

This is a description of a posterior cord lesion. Remember that the posterior cord

gives rise to the axillary and radial nerve.

Cords of the brachial plexus

The brachial plexus cords are described according to their relationship with the

axillary artery. The cords pass over the 1st rib near to the dome of the lung and pass

beneath the clavicle immediately posterior to the subclavian artery.

Lateral cord

Anterior divisions of the upper and middle trunks form the lateral cord

Origin of the lateral pectoral nerve (C5, C6, C7)

Medial cord

Anterior division of the lower trunk forms the medial cord

Origin of the medial pectoral nerve (C8, T1), the medial brachial cutaneous

nerve (T1), and the medial antebrachial cutaneous nerve (C8, T1)

Posterior cord

Formed by the posterior divisions of the 3 trunks (C5-T1)

Page 172: MRCS preparation emrcs questions upperlimb

Origin of the upper and lower subscapular nerves (C7, C8 and C5, C6,

respectively) and the thoracodorsal nerve to the latissimus dorsi (also known

as the middle subscapular nerve, C6, C7, C8), axillary and radial nerve

A motor cyclist is involved in a road traffic accident causing severe right shoulder injuries. He

is found to have an adducted, medially rotated shoulder. The elbow is fully extended and the

forearm pronated. Which is the most likely diagnosis?

A. C8, T1 root lesion

B. C5, C6 root lesion

C. Radial nerve lesion

D. Ulnar nerve lesion

E. Axillary nerve lesion

Erbs Palsy C5, C6 lesion

The features include:

Waiter's tip position Loss of shoulder abduction (deltoid and supraspinatus paralysis) Loss of external rotation of the shoulder (paralysis of infraspinatus and teres major) Loss of elbow flexion (paralysis of biceps, brachialis and brachioradialis) Loss of forearm supination (paralysis of Biceps)

The motorcyclist has had an Erb's palsy (C5, C6 root lesion). This is commonly known to be

associated with birth injury when a baby has a shoulder dystocia.

Brachial plexus

Origin Anterior rami of C5 to T1

Sections of the

plexus

Roots, trunks, divisions, cords, branches Mnemonic:Real Teenagers Drink Cold Beer

Roots Located in the posterior triangle Pass between scalenus anterior and medius

Page 173: MRCS preparation emrcs questions upperlimb

Trunks Located posterior to middle third of clavicle Upper and middle trunks related superiorly to the subclavian

artery Lower trunk passes over 1st rib posterior to the subclavian

artery

Divisions Apex of axilla

Cords Related to axillary artery

Diagram illustrating the branches of the brachial plexus

Image sourced from Wikipedia

Cutaneous sensation of the upper limb

Page 174: MRCS preparation emrcs questions upperlimb

Image sourced from Wikipedia

A 23 year old man has a cannula inserted into his cephalic vein. Through which

structure does the cephalic vein pass?

A. Interosseous membrane

B. Triceps

C. Pectoralis major

D. Clavipectoral fascia

E. Tendon of biceps

The cephalic vein is a favored vessel for arteriovenous fistula formation and should be

preserved in patients with end stage renal failure

The cephalic vein penetrates the calvipectoral fascia (but not the pectoralis major)

prior to terminating in the axillary vein.

Cephalic vein

Path

Dorsal venous arch drains laterally into the cephalic vein

Page 175: MRCS preparation emrcs questions upperlimb

Crosses the anatomical snuffbox and travels laterally up the arm

At the antecubital fossa connected to the basilic vein by the median cubital

vein

Pierces deep fascia of deltopectoral groove to join axillary vein

Rate questio

Which of the following is not a muscle of the rotator cuff?

A. Subscapularis

B. Teres minor

C. Supraspinatus

D. Infraspinatus

E. Deltoid

Deltoid may abduct the shoulder and is not a rotator cuff muscle.

Muscles of the rotator cuff

Muscle Innervation

Supraspinatus muscle Suprascapular nerve

Infraspinatus muscle Suprascapular nerve

Teres minor muscle Axillary nerve

Subscapularis muscle Superior and inferior subscapular nerves

A 32 year old man is stabbed in the neck and the inferior trunk of his brachial plexus is

injured. Which of the modalities listed below is least likely to be affected?

A. Initiating abduction of the shoulder

B. Abduction of the fingers

C. Flexion of the little finger

D. Sensation on the palmar aspect of the little finger

E. Gripping a screwdriver

Page 176: MRCS preparation emrcs questions upperlimb

Inferior trunk of brachial plexus.

C8 and T1 roots Contributes to ulnar nerve and

part of median nerve

Theme from September 2012 Exam

The inferior trunk of the brachial plexus is rarely injured. Nerve roots C8 and T1 are the main

contributors to this trunk. Therefore an injury to this site will most consistently affect the

ulnar nerve. The inferior trunk also contributes to the median nerve by way of the posterior

division and therefore some impairment of grip is almost inevitable.

Brachial plexus

Origin Anterior rami of C5 to T1

Sections of the

plexus

Roots, trunks, divisions, cords, branches Mnemonic:Real Teenagers Drink Cold Beer

Roots Located in the posterior triangle Pass between scalenus anterior and medius

Trunks Located posterior to middle third of clavicle Upper and middle trunks related superiorly to the subclavian

artery Lower trunk passes over 1st rib posterior to the subclavian

artery

Divisions Apex of axilla

Cords Related to axillary artery

Diagram illustrating the branches of the brachial plexus

Page 177: MRCS preparation emrcs questions upperlimb

Image sourced from Wikipedia

Cutaneous sensation of the upper limb

Image sourced from Wikipedia

As it exits the axilla the radial nerve lies on which of the following muscles?

Page 178: MRCS preparation emrcs questions upperlimb

A. Supraspinatus

B. Infraspinatus

C. Teres major

D. Deltoid

E. Pectoralis major

The radial nerve passes through the triangular space to leave the axilla. The superior border

of this is bounded by the teres major muscle to which the radial nerve is closely related.

Radial nerve

Continuation of posterior cord of the brachial plexus (root values C5 to T1)

Path

In the axilla: lies posterior to the axillary artery on subscapularis, latissimus dorsi and teres major.

Enters the arm between the brachial artery and the long head of triceps (medial to humerus).

Spirals around the posterior surface of the humerus in the groove for the radial nerve.

At the distal third of the lateral border of the humerus it then pierces the intermuscular septum and descends in front of the lateral epicondyle.

At the lateral epicondyle it lies deeply between brachialis and brachioradialis where it then divides into a superficial and deep terminal branch.

Deep branch crosses the supinator to become the posterior interosseous nerve.

In the image below the relationships of the radial nerve can be appreciated

Page 179: MRCS preparation emrcs questions upperlimb

Image sourced from Wikipedia

Regions innervated

Motor (main nerve) Triceps Anconeus Brachioradialis Extensor carpi radialis

Motor (posterior

interosseous

branch

Extensor carpi ulnaris Extensor digitorum Extensor indicis Extensor digiti minimi Extensor pollicis longus and brevis Abductor pollicis longus

Sensory The area of skin supplying the proximal phalanges on the dorsal aspect

of the hand is supplied by the radial nerve (this does not apply to the

little finger and part of the ring finger)

Muscular innervation and effect of denervation

Anatomical

location

Muscle affected Effect of paralysis

Shoulder Long head of triceps Minor effects on shoulder stability in

abduction

Page 180: MRCS preparation emrcs questions upperlimb

Arm Triceps Loss of elbow extension

Forearm Supinator

Brachioradialis

Extensor carpi radialis

longus and brevis

Weakening of supination of prone hand and

elbow flexion in mid prone position

The cutaneous sensation of the upper limb- illustrating the contribution of the radial nerve

Image sourced from Wikipedia

A 62 year old man presents with arm weakness. On examination he has a weakness of elbow

extension and loss of sensation on the dorsal aspect of the first digit. What is the site of the

most likely underlying defect?

A. Axillary nerve

B. Median nerve

C. Ulnar nerve

Page 181: MRCS preparation emrcs questions upperlimb

D. Radial nerve

E. Musculocutaneous nerve

Theme from April 2012 Exam

The long head of the triceps muscle may be innervated by the axillary nerve and therefore

complete loss of triceps muscles function may not be present even with proximally sited

nerve lesions.

Radial nerve

Continuation of posterior cord of the brachial plexus (root values C5 to T1)

Path

In the axilla: lies posterior to the axillary artery on subscapularis, latissimus dorsi and teres major.

Enters the arm between the brachial artery and the long head of triceps (medial to humerus).

Spirals around the posterior surface of the humerus in the groove for the radial nerve.

At the distal third of the lateral border of the humerus it then pierces the intermuscular septum and descends in front of the lateral epicondyle.

At the lateral epicondyle it lies deeply between brachialis and brachioradialis where it then divides into a superficial and deep terminal branch.

Deep branch crosses the supinator to become the posterior interosseous nerve.

In the image below the relationships of the radial nerve can be appreciated

Page 182: MRCS preparation emrcs questions upperlimb

Image sourced from Wikipedia

Regions innervated

Motor (main nerve) Triceps Anconeus Brachioradialis Extensor carpi radialis

Motor (posterior

interosseous

branch

Extensor carpi ulnaris Extensor digitorum Extensor indicis Extensor digiti minimi Extensor pollicis longus and brevis Abductor pollicis longus

Sensory The area of skin supplying the proximal phalanges on the dorsal aspect

of the hand is supplied by the radial nerve (this does not apply to the

little finger and part of the ring finger)

Muscular innervation and effect of denervation

Anatomical

location

Muscle affected Effect of paralysis

Shoulder Long head of triceps Minor effects on shoulder stability in

abduction

Page 183: MRCS preparation emrcs questions upperlimb

Arm Triceps Loss of elbow extension

Forearm Supinator

Brachioradialis

Extensor carpi radialis

longus and brevis

Weakening of supination of prone hand and

elbow flexion in mid prone position

The cutaneous sensation of the upper limb- illustrating the contribution of the radial nerve

Im

From which of the following structures does the long head of the triceps muscle arise?

A. Coracoid process

B. Acromion

C. Infraglenoid tubercle

D. Coraco-acromial ligament

E. Coraco-humeral ligament

The long head arises from the infraglenoid tubercle. The fleshy lateral and medial

Page 184: MRCS preparation emrcs questions upperlimb

heads are attached to the posterior aspect of the humerus between the insertion of the

teres minor and the olecranon fossa.

Triceps

Origin Long head- infraglenoid tubercle of the scapula.

Lateral head- dorsal surface of the humerus, lateral and proximal

to the groove of the radial nerve

Medial head- posterior surface of the humerus on the

inferomedial side of the radial groove and both of the

intermuscular septae

Insertion Olecranon process of the ulna. Here the olecranon bursa is

between the triceps tendon and olecranon.

Some fibres insert to the deep fascia of the forearm, posterior

capsule of the elbow (preventing the capsule from being trapped

between olecranon and olecranon fossa during extension)

Innervation Radial nerve

Blood

supply

Profunda brachii artery

Action Elbow extension. The long head can adduct the humerus and and extend

it from a flexed position

Relations The radial nerve and profunda brachii vessels lie between the lateral and

medial heads

A 58 year old lady presents with a mass in the upper outer quadrant of the right breast.

Which of the following statements relating to the breast is untrue?

A. The internal mammary artery provides the majority of its arterial supply

B. Nipple retraction may occur as a result of tumour infiltration of the

clavipectoral fascia

C. The internal mammary artery is a branch of the subclavian artery

D. Up to 70% of lymphatic drainage is to the ipsilateral axillary nodes

E. None of the above

Nipple retraction is a feature of breast malignancy. However, it is typically caused by tumour

infiltration of Coopers Ligaments that run through the breast and surround the lobules. The

clavipectoral fascia encases the axillary contents. The lymphatic drainage of the breast is to

Page 185: MRCS preparation emrcs questions upperlimb

the axilla and also to the internal mammary chain. The breast is well vascularised and the

internal mammary artery is a branch of the subclavian artery.

Breast

The breast itself lies on a layer of pectoral fascia and the following muscles:

1. Pectoralis major

2. Serratus anterior

3. External oblique

Image showing the topography of the female breast

Image sourced from Wikipedia

Breast anatomy

Nerve supply Branches of intercostal nerves from T4-T6.

Arterial supply Internal mammary (thoracic) artery External mammary artery (laterally) Anterior intercostal arteries Thoraco-acromial artery

Venous drainage Superficial venous plexus to sub clavian, axillary and intercostal veins.

Lymphatic

drainage

70% Axillary nodes Internal mammary chain Other lymphatic sites such as deep cervical and supraclavicular

Page 186: MRCS preparation emrcs questions upperlimb

fossa (later in disease)

Theme: Nerve injury

A. Median nerve

B. Ulnar nerve

C. Radial nerve

D. Anterior interosseous nerve

E. Posterior interosseous nerve

F. Axillary nerve

G. Musculocutaneous nerve

Please select the nerve at risk of injury in each scenario. Each option may be used once,

more than once or not at all.

290. A 43 year old typist presents with pain at the dorsal aspect of the upper part of her

forearm. She also complains of weakness when extending her fingers. On

examination triceps and supinator are both functioning normally. There is weakness

of most of the extensor muscles. However, there is no sensory deficit.

Posterior interosseous nerve

The radial nerve may become entrapped in the "arcade of Frohse" which is a

superficial part of the supinator muscle which overlies the posterior interosseous

nerve. This nerve is entirely muscular and articular in its distribution. It passes

postero-inferiorly and gives branches to extensor carpi radialis brevis and

supinator. It enters supinator and curves around the lateral and posterior surfaces

of the radius. On emerging from the supinator the posterior interosseous nerve lies

between the superficial extensor muscles and the lowermost fibres of supinator. It

then gives branches to the extensors.

291. A 28 year teacher reports difficulty with writing. There is no sensory loss. She is

known to have an aberrant Gantzer muscle.

You answered Posterior interosseous nerve

Page 187: MRCS preparation emrcs questions upperlimb

The correct answer is Anterior interosseous nerve

Anterior interosseous lesions occur due to fracture, or rarely due to compression.

The Gantzer muscle is an aberrant accessory of the flexor pollicis longus and is a risk

factor for anterior interosseous nerve compression. Remember loss of pincer grip

and normal sensation indicates an interosseous nerve lesion.

292. A 35 year tennis player attends reporting tingling down his arm. He says that his

'funny bone' was hit very hard by a tennis ball. There is weakness of abduction and

adduction of his extended fingers.

Ulnar nerve

Theme from September 2012 exam

The ulnar nerve arises from the medial cord of the brachial plexus (C8, T1 and

contribution from C7). The nerve descends between the axillary artery and vein,

posterior to the cutaneous nerve of the forearm and then lies anterior to triceps on

the medial side of the brachial artery. In the distal half of the arm it passes through

the medial intermuscular septum, and continues between this structure and the

medial head of triceps to enter the forearm between the medial epicondyle of the

humerus and the olecranon. It may be injured at this site in this scenario.

Brachial plexus

Origin Anterior rami of C5 to T1

Sections of the

plexus

Roots, trunks, divisions, cords, branches Mnemonic:Real Teenagers Drink Cold Beer

Roots Located in the posterior triangle Pass between scalenus anterior and medius

Trunks Located posterior to middle third of clavicle Upper and middle trunks related superiorly to the subclavian

artery Lower trunk passes over 1st rib posterior to the subclavian

artery

Page 188: MRCS preparation emrcs questions upperlimb

Divisions Apex of axilla

Cords Related to axillary artery

Diagram illustrating the branches of the brachial plexus

Image sourced from Wikipedia

Cutaneous sensation of the upper limb

Page 189: MRCS preparation emrcs questions upperlimb

Image sourced from Wikipedia

A 53 year old lady presents with pain and discomfort in her hand. She works as a typist and

notices that the pain is worst when she is working. She also suffers symptoms at night. Her

little finger is less affected by the pain. Which of the nerves listed below is most likely to be

affected?

A. Radial

B. Median

C. Ulnar

D. Anterior interosseous nerve

E. Posterior interosseous nerve

Motor supply: LOAF

L ateral 2 lumbricals

O pponens pollicis

Page 190: MRCS preparation emrcs questions upperlimb

A bductor

pollicisbrevis

F lexor pollicis

brevis

Theme from April 2012 Exam

The most likely diagnosis here is carpal tunnel syndrome, the median nerve is compressed in

the wrist and symptoms usually affect the fingers and wrist either at night or when the hand

is being used (e.g. as a typist).

Median nerve

The median nerve is formed by the union of a lateral and medial root respectively from the

lateral (C5,6,7) and medial (C8 and T1) cords of the brachial plexus; the medial root passes

anterior to the third part of the axillary artery. The nerve descends lateral to the brachial

artery, crosses to its medial side (usually passing anterior to the artery). It passes deep to the

bicipital aponeurosis and the median cubital vein at the elbow.

It passes between the two heads of the pronator teres muscle, and runs on the deep surface

of flexor digitorum superficialis (within its fascial sheath).

Near the wrist it becomes superficial between the tendons of flexor digitorum superficialis

and flexor carpi radialis, deep to palmaris longus tendon. It passes deep to the flexor

retinaculum to enter the palm, but lies anterior to the long flexor tendons within the carpal

tunnel.

Branches

Region Branch

Upper arm No branches, although the nerve commonly communicates with the

musculocutaneous nerve

Forearm Pronator teres

Flexor carpi radialis

Palmaris longus

Flexor digitorum superficialis

Flexor pollicis longus

Flexor digitorum profundus (only the radial half)

Distal

forearm

Palmar cutaneous branch

Hand

(Motor)

Motor supply (LOAF)

Lateral 2 lumbricals Opponens pollicis

Page 191: MRCS preparation emrcs questions upperlimb

Abductor pollicis brevis Flexor pollicis brevis

Hand

(Sensory)

Over thumb and lateral 2 ½ fingers On the palmar aspect this projects proximally, on the dorsal aspect only

the distal regions are innervated with the radial nerve providing the more proximal cutaneous innervation.

Patterns of damage

Damage at wrist

e.g. carpal tunnel syndrome paralysis and wasting of thenar eminence muscles and opponens pollicis (ape hand

deformity) sensory loss to palmar aspect of lateral (radial) 2 ½ fingers

Damage at elbow, as above plus:

unable to pronate forearm weak wrist flexion ulnar deviation of wrist

Anterior interosseous nerve (branch of median nerve)

leaves just below the elbow results in loss of pronation of forearm and weakness of long flexors of thumb and

index finger

Topography of the median nerve

Page 192: MRCS preparation emrcs questions upperlimb

Image sourced from Wikipedia

A 24 year female is admitted to A&E with tingling of her hand after a fall. She is found to

have a fracture of the medial epicondyle. What is the most likely nerve lesion?

A. Ulnar nerve

B. Radial nerve

C. Median nerve

D. Axillary nerve

E. Cutaneous nerve

Page 193: MRCS preparation emrcs questions upperlimb

The radial nerve is located near the lateral epicondyle.

Ulnar nerve

Origin

C8, T1

Supplies (no muscles in the upper arm)

Flexor carpi ulnaris Flexor digitorum profundus Flexor digiti minimi Abductor digiti minimi Opponens digiti minimi Adductor pollicis Interossei muscle Third and fourth lumbricals Palmaris brevis

Path

Posteromedial aspect of ulna to flexor compartment of forearm, then along the ulnar. Passes beneath the flexor carpi ulnaris muscle, then superficially through the flexor retinaculum into the palm of the hand.

Page 194: MRCS preparation emrcs questions upperlimb

Image sourced from Wikipedia

Branches

Branch Supplies

Articular branch Flexor carpi ulnaris

Medial half of the flexor digitorum profundus

Palmar cutaneous branch (Arises near the

middle of the forearm)

Skin on the medial part of the palm

Dorsal cutaneous branch Dorsal surface of the medial part of the hand

Superficial branch Cutaneous fibres to the anterior surfaces of the

medial one and one-half digits

Deep branch Hypothenar muscles

All the interosseous muscles

Third and fourth lumbricals

Page 195: MRCS preparation emrcs questions upperlimb

Adductor pollicis

Medial head of the flexor pollicis brevis

Effects of injury

Damage at the wrist Wasting and paralysis of intrinsic hand muscles (claw hand) Wasting and paralysis of hypothenar muscles Loss of sensation medial 1 and half fingers

Damage at the

elbow

Radial deviation of the wrist Clawing less in 3rd and 4th digits

A 43 year old lady is undergoing an axillary node clearance for breast cancer. The

nodal disease is bulky. During clearance of the level 3 nodes there is suddenly brisk

haemorrhage. The most likely vessel responsible is:

A. Thoracoacromial artery

B. Cephalic vein

C. Thoracodorsal trunk

D. Internal mammary artery

E. Posterior circumflex humeral artery

The thoracoacromial artery pierces the pectoralis major and gives off branches within

this space. The level 3 axillary nodes lie between pectoralis major and

minor.Although the thoracodorsal trunk may be injured during an axillary dissection it

does not lie within the level 3 nodes.

Thoracoacromial artery

The thoracoacromial artery (acromiothoracic artery; thoracic axis) is a short trunk,

which arises from the forepart of the axillary artery, its origin being generally

overlapped by the upper edge of the Pectoralis minor.

Projecting forward to the upper border of the Pectoralis minor, it pierces the

coracoclavicular fascia and divides into four branches: pectoral, acromial, clavicular,

and deltoid.

Branch Description

Pectoral

branch

Descends between the two Pectoral muscles, and is distributed to them

and to the breast, anastomosing with the intercostal branches of the

internal thoracic artery and with the lateral thoracic.

Page 196: MRCS preparation emrcs questions upperlimb

Acromial

branch

Runs laterally over the coracoid process and under the Deltoid, to which

it gives branches; it then pierces that muscle and ends on the acromion in

an arterial network formed by branches from the suprascapular,

thoracoacromial, and posterior humeral circumflex arteries.

Clavicular

branch

Runs upwards and medially to the sternoclavicular joint, supplying this

articulation, and the Subclavius

Deltoid

branch

Arising with the acromial, it crosses over the Pectoralis minor and passes

in the same groove as the cephalic vein, between the Pectoralis major and

Deltoid, and gives branches to both muscles.

A 73 year old lady with long standing atrial fibrillation develops a cold and pulseless

white arm. A brachial embolus is suspected and a brachial embolectomy is performed.

Which of the following structures is at greatest risk of injury during this procedure?

A. Radial nerve

B. Cephalic vein

C. Ulnar nerve

D. Median nerve

E. None of the above

The median nerve lies close to the brachial artery in the antecubital fossa. This is the

usual site of surgical access to the brachial artery for an embolectomy procedure. The

median nerve may be damaged during clumsy application of vascular clamps to the

artery.

Brachial artery

The brachial artery begins at the lower border of teres major as a continuation of the

axillary artery. It terminates in the cubital fossa at the level of the neck of the radius

by dividing into the radial and ulnar arteries.

Relations Posterior relations include the long head of triceps with the radial nerve and profunda

vessels intervening. Anteriorly it is overlapped by the medial border of biceps.

It is crossed by the median nerve in the middle of the arm.

In the cubital fossa it is separated from the median cubital vein by the bicipital

aponeurosis.

The basilic vein is in contact at the most proximal aspect of the cubital fossa and lies

medially. Which of the following fingers is not a point of attachment for the palmar interossei?

A. Middle finger

Page 197: MRCS preparation emrcs questions upperlimb

B. Little finger

C. Ring finger

D. Index finger

E. None of the above

The middle finger has no attachment of the palmar interosseous.

Image sourced from Wikipedia

Hand

Anatomy of the hand

Bones 8 Carpal bones 5 Metacarpals 14 phalanges

Page 198: MRCS preparation emrcs questions upperlimb

Intrinsic Muscles 7 Interossei - Supplied by ulnar nerve

3 palmar-adduct fingers 4 dorsal- abduct fingers

Intrinsic muscles Lumbricals

Flex MCPJ and extend the IPJ. Origin deep flexor tendon and insertion dorsal extensor hood

mechanism. Innervation: 1st and 2nd- median nerve, 3rd and 4th- deep

branch of the ulnar nerve.

Thenar eminence Abductor pollicis brevis Opponens pollicis Flexor pollicis brevis

Hypothenar

eminence

Opponens digiti minimi Flexor digiti minimi brevis Abductor digiti minimi

Image sourced from Wikipedia

Page 199: MRCS preparation emrcs questions upperlimb

A 6 year old sustains a supracondylar fracture of the distal humerus. There are concerns that

the radial nerve may have been injured. What is the relationship of the radial nerve to the

humerus at this point?

A. Anterolateral

B. Anteromedial

C. Posterolateral

D. Posteromedial

E. Immediately anterior

The radial nerve lies anterolateral to the humerus in the supracondylar area.

Radial nerve

Continuation of posterior cord of the brachial plexus (root values C5 to T1)

Path

In the axilla: lies posterior to the axillary artery on subscapularis, latissimus dorsi and teres major.

Enters the arm between the brachial artery and the long head of triceps (medial to humerus).

Spirals around the posterior surface of the humerus in the groove for the radial nerve.

At the distal third of the lateral border of the humerus it then pierces the intermuscular septum and descends in front of the lateral epicondyle.

At the lateral epicondyle it lies deeply between brachialis and brachioradialis where it then divides into a superficial and deep terminal branch.

Deep branch crosses the supinator to become the posterior interosseous nerve.

In the image below the relationships of the radial nerve can be appreciated

Page 200: MRCS preparation emrcs questions upperlimb

Image sourced from Wikipedia

Regions innervated

Motor (main nerve) Triceps Anconeus Brachioradialis Extensor carpi radialis

Motor (posterior

interosseous

branch

Extensor carpi ulnaris Extensor digitorum Extensor indicis Extensor digiti minimi Extensor pollicis longus and brevis Abductor pollicis longus

Sensory The area of skin supplying the proximal phalanges on the dorsal aspect

of the hand is supplied by the radial nerve (this does not apply to the

little finger and part of the ring finger)

Muscular innervation and effect of denervation

Anatomical

location

Muscle affected Effect of paralysis

Shoulder Long head of triceps Minor effects on shoulder stability in

abduction

Page 201: MRCS preparation emrcs questions upperlimb

Arm Triceps Loss of elbow extension

Forearm Supinator

Brachioradialis

Extensor carpi radialis

longus and brevis

Weakening of supination of prone hand and

elbow flexion in mid prone position

The cutaneous sensation of the upper limb- illustrating the contribution of the radial nerve

Image sourced from Wikipedia

The following are true of the ulnar nerve except:

A. It innervates the palmar interossei

B. Derived from the medial cord of the brachial plexus

C. Supplies the muscles of the thenar eminence

D. Supplies the medial half of flexor digitorum profundus

Page 202: MRCS preparation emrcs questions upperlimb

E. Passes superficial to the flexor retinaculum

These are supplied by the median nerve and atrophy of these is a feature of carpal tunnel

syndrome

Ulnar nerve

Origin

C8, T1

Supplies (no muscles in the upper arm)

Flexor carpi ulnaris Flexor digitorum profundus Flexor digiti minimi Abductor digiti minimi Opponens digiti minimi Adductor pollicis Interossei muscle Third and fourth lumbricals Palmaris brevis

Path

Posteromedial aspect of ulna to flexor compartment of forearm, then along the ulnar. Passes beneath the flexor carpi ulnaris muscle, then superficially through the flexor retinaculum into the palm of the hand.

Page 203: MRCS preparation emrcs questions upperlimb

Image sourced from Wikipedia

Branches

Branch Supplies

Articular branch Flexor carpi ulnaris

Medial half of the flexor digitorum profundus

Palmar cutaneous branch (Arises near the

middle of the forearm)

Skin on the medial part of the palm

Dorsal cutaneous branch Dorsal surface of the medial part of the hand

Superficial branch Cutaneous fibres to the anterior surfaces of the

medial one and one-half digits

Deep branch Hypothenar muscles

All the interosseous muscles

Third and fourth lumbricals

Page 204: MRCS preparation emrcs questions upperlimb

Adductor pollicis

Medial head of the flexor pollicis brevis

Effects of injury

Damage at the wrist Wasting and paralysis of intrinsic hand muscles (claw hand) Wasting and paralysis of hypothenar muscles Loss of sensation medial 1 and half fingers

Damage at the

elbow

Radial deviation of the wrist Clawing less in 3rd and 4th digits

Which muscle is responsible for causing flexion of the distal interphalangeal joint of the ring

finger?

A. Flexor digitorum superficialis

B. Lumbricals

C. Palmar interossei

D. Flexor digitorum profundus

E. Flexor digiti minimi brevis

Flexor digitorum superficialis and flexor digitorum profundus are responsible for causing

flexion. The superficialis tendons insert on the bases of the middle phalanges; the profundus

tendons insert on the bases of the distal phalanges. Both tendons flex the wrist, MCP and

PIP joints; however, only the profundus tendons flex the DIP joints.

Hand

Anatomy of the hand

Bones 8 Carpal bones 5 Metacarpals 14 phalanges

Intrinsic Muscles 7 Interossei - Supplied by ulnar nerve

3 palmar-adduct fingers

Page 205: MRCS preparation emrcs questions upperlimb

4 dorsal- abduct fingers

Intrinsic muscles Lumbricals

Flex MCPJ and extend the IPJ. Origin deep flexor tendon and insertion dorsal extensor hood

mechanism. Innervation: 1st and 2nd- median nerve, 3rd and 4th- deep

branch of the ulnar nerve.

Thenar eminence Abductor pollicis brevis Opponens pollicis Flexor pollicis brevis

Hypothenar

eminence

Opponens digiti minimi Flexor digiti minimi brevis Abductor digiti minimi

Image sourced from Wikipedia

Which of the following muscles lies medial to the long thoracic nerve?

A. Serratus anterior

Page 206: MRCS preparation emrcs questions upperlimb

B. Latissimus dorsi

C. Pectoralis major

D. Pectoralis minor

E. None of the above

Theme from 2009 Exam

Long thoracic nerve

Derived from ventral rami of C5, C6, and C7 (close to their emergence from intervertebral

foramina)

It runs downward and passes either anterior or posterior to the middle scalene muscle

It reaches upper tip of serratus anterior muscle and descends on outer surface of this

muscle, giving branches into it

Winging of Scapula occurs in long thoracic nerve injury (most common) or from spinal

accessory nerve injury (which denervates the trapezius) or a dorsal scapular nerve injury